New classes are starting soon - secure your spot today!

2009 Alternate AIME Math Jam

Go back to the Math Jam Archive

AoPS instructors will lead a discussion on all 15 problems from the 2009 Alternate AIME.

Copyright © 2024 AoPS Incorporated. This page is copyrighted material. You can view and print this page for your own use, but you cannot share the contents of this file with others.

Facilitator: Richard Rusczyk

rrusczyk 2009-04-03 18:59:49
Hello and welcome to the 2009 AIME II Math Jam!
rrusczyk 2009-04-03 18:59:56
Before we get started I would like to take a moment to explain our virtual classroom to those who have not previously participated in a Math Jam or one of our online classes
rrusczyk 2009-04-03 19:00:04
The classroom is moderated, meaning that students can type into the classroom, but only the moderator can choose a comment to drop into the classroom. This helps keep the class organized and on track. This also means that only well-written comments will be dropped into the classroom, so please take time writing responses that are complete and easy to read.
rrusczyk 2009-04-03 19:00:17
There are a lot of students here! As I said, only well-written comments will be passed to the entire group. Please do not take it personally if your comments do not get posted, and please do not complain about it. I expect this Math Jam to be much larger than our typical class, so please be patient with me---there are quite a few of you here tonight!
rrusczyk 2009-04-03 19:00:32
Also, we won't be going through the math quite as thoroughly as we do in our classes -- I can't teach all the prerequisite material for every problem as we go (and certainly not to 200+ students -- this Math Jam won't be as orderly or as thorough as our typical online classes.)! If you do not know a particular concept necessary for a problem, I probably will not be able to explain it to you tonight.
rrusczyk 2009-04-03 19:01:08
(I don't think we'll get 200+ students tonight like we had for the first AMC Math Jam. Phew.)
rrusczyk 2009-04-03 19:01:18
Please also remember that the purpose of this Math Jam is to work through the solutions to the AIME problems, and not to merely present the answers. "Working through the solutions" includes discussing problem-solving tactics. So please, when a problem is posted, do not simply respond with the answer. That's not why we're here. We're going to work through the problems step-by-step, and people who post comments that skip key steps or jump ahead in the problem, without providing explanation or motivation, are going to be ignored.
rrusczyk 2009-04-03 19:01:59
I don't claim that the solutions that we'll present today are the "definitive" or "best" solutions to each problem. Many of the problems have more than one plausible approach. We don't have time to consider every possible approach to every problem, but you can discuss other approaches on the Forum or post them in the AoPSWiki.
rrusczyk 2009-04-03 19:02:12
Our agenda is to work through all 15 problems on the 2009 AIME II, in order.
rrusczyk 2009-04-03 19:02:30
Let's get to the problems!
rrusczyk 2009-04-03 19:02:32
rrusczyk 2009-04-03 19:03:07
Here are a couple ways to tackle the problem:
Recursive Acronym 2009-04-03 19:03:12
Since you need the white, red, and blue paint levels to be equal *before* painting the white, red, and blue stripes, you need to use up enough paint in the pink stripe to bring all three levels to 130.
GoldenFrog1618 2009-04-03 19:03:12
let x be the amount of paint for each stripe
Hullohihell0 2009-04-03 19:03:12
set up an equation
rrusczyk 2009-04-03 19:03:33
We could set up an equation and do some algebra. Or, we can reason our way to the answer pretty quickly.
rrusczyk 2009-04-03 19:03:43
Let's look at Recursive's solution first:
Recursive Acronym 2009-04-03 19:03:45
So you use 34 ounces of red and 58 ounces of white on the pink.
rrusczyk 2009-04-03 19:04:13
We use 164 - 130 = 34 more ounces of red and 188 - 130 = 58 more ounces of white paint in making the stripes, since in the end we must have the same amount of all three colors.
Recursive Acronym 2009-04-03 19:04:22
Which means 92 ounces a stripe.
GoldenFrog1618 2009-04-03 19:04:22
92 ounces per stripe
rrusczyk 2009-04-03 19:04:34
So there's a total of 34 + 58 = 92 ounces in the pink stripe
tinytim 2009-04-03 19:04:38
Thus, you use 92 ounces of blue because the stripes are the same size
Recursive Acronym 2009-04-03 19:05:00
So then you use 92 on the pink stripe, and are left with 130 of each, which means that after using 92 on Red, White, and Blue, you have 38 left on each stripe.
rrusczyk 2009-04-03 19:05:03
Therefore, there are 92 ounces of paint in the blue stripe, leaving 130 - 92 = 38 ounces of blue paint.
GoldenFrog1618 2009-04-03 19:05:22
130*3-92*3=114
mathspee 2009-04-03 19:05:22
so there are 114 ounces left
geoffreywong 2009-04-03 19:05:22
so subtract 130-92 and multiply by three
Recursive Acronym 2009-04-03 19:05:22
38 ounces left of each color times 3 colors equals 114 ounces left.
tinytim 2009-04-03 19:05:22
Which gives an answer of 38*3=114 ounces
rrusczyk 2009-04-03 19:05:29
This means we have 3(38) = 114 ounces of paint left over.
rrusczyk 2009-04-03 19:05:37
We also could have tackled this with algebra.
rrusczyk 2009-04-03 19:05:58
We let the amount of paint in each stripe be t.
rrusczyk 2009-04-03 19:06:12
Then what?
GoldenFrog1618 2009-04-03 19:06:52
(188-t)+(164-t)-t=2(130-t)
tinytim 2009-04-03 19:07:04
2(180-t)=(164-t)+(188-t)-t
rrusczyk 2009-04-03 19:07:08
Where does this come from?
tinytim 2009-04-03 19:07:18
130 I mean
tinytim 2009-04-03 19:07:38
We know that the same amount of each paint are left
rrusczyk 2009-04-03 19:07:42
Then, we have used t ounces of blue, so there is 130-t blue left.
chessmaster7 2009-04-03 19:07:45
the stripes are all the same size, so two stripes should equal two stripes
rrusczyk 2009-04-03 19:07:49
We must also have 130-t red and 130-t white, for a total of 260-2t.
GoldenFrog1618 2009-04-03 19:07:51
t ounces is removed from each stripe and another t ounces is removed from the red and white
MathWhiz2009 2009-04-03 19:08:04
2(130-t)=188+164-t-t-t
rrusczyk 2009-04-03 19:08:06
We started with 164 + 188 = 352 ounces of red and white combined, and used 3t total on the red, white, and pink stripes, so we have 352 - 3t left.
rrusczyk 2009-04-03 19:08:10
Therefore, we must have 352 - 3t = 260 - 2t.
GoldenFrog1618 2009-04-03 19:08:28
t=92
tinytim 2009-04-03 19:08:28
so t=92 as we earlier found
mathq 2009-04-03 19:08:37
t=352-260=92
saszs 2009-04-03 19:08:40
t=92
chessmaster7 2009-04-03 19:08:49
this provides t=92, and 130*3-92*3=114
MathWhiz2009 2009-04-03 19:08:49
then solve the equation to find t, then subtract 4t from 130+164+188
GoldenFrog1618 2009-04-03 19:08:49
188+130+164-92*4=114
rrusczyk 2009-04-03 19:08:50
From here, we find t=92, just like before, and we have a total of 3(130-t) = 114 ounces left.
rrusczyk 2009-04-03 19:09:01
On to #2!
rrusczyk 2009-04-03 19:09:02
rrusczyk 2009-04-03 19:09:11
There sure doesn't look like any easy way to combine the three numbers we have to add, so let's see if we can compute one individually.
rrusczyk 2009-04-03 19:09:18
Recursive Acronym 2009-04-03 19:10:02
Rewrite x^(y^2) as (x^y)^y
dgreenb801 2009-04-03 19:10:02
a^{x^2}=(a^x)^x
GoldenFrog1618 2009-04-03 19:10:02
Hyperboloid 2009-04-03 19:10:02
rrusczyk 2009-04-03 19:10:17
rrusczyk 2009-04-03 19:10:19
Then what?
spelljack825 2009-04-03 19:10:47
substitute
mathq 2009-04-03 19:10:48
then it becomes 27^log7(base 3)
chessmaster7 2009-04-03 19:10:48
then substitute in 27 for 27^(log_3(7))
rrusczyk 2009-04-03 19:10:53
rrusczyk 2009-04-03 19:11:06
All we did here was substitute the given information.
rrusczyk 2009-04-03 19:11:12
Can we simplify this further?
GoldenFrog1618 2009-04-03 19:11:34
27^(log_3 7)=7^3=343
spelljack825 2009-04-03 19:11:34
27 = 3^3
m1sterzer0 2009-04-03 19:11:34
27=3^3
tinytim 2009-04-03 19:11:34
3 to "something" equals 7. Thus, 27 to that "something equals 7^3=343.
maxschindler 2009-04-03 19:11:34
3^3log_3{7}
rrusczyk 2009-04-03 19:11:52
rrusczyk 2009-04-03 19:12:01
tinytim 2009-04-03 19:12:18
11^2
Hyperboloid 2009-04-03 19:12:18
121
GoldenFrog1618 2009-04-03 19:12:18
121
rrusczyk 2009-04-03 19:12:21
rrusczyk 2009-04-03 19:12:39
This is basically the same thing we did for the first one.
rrusczyk 2009-04-03 19:12:43
sj0201 2009-04-03 19:12:54
then 5
tinytim 2009-04-03 19:12:54
25^(1/2)=5
kohjhsd 2009-04-03 19:12:54
25^.5
nechesskid13 2009-04-03 19:12:54
5
GoldenFrog1618 2009-04-03 19:12:54
e714212835 2009-04-03 19:12:54
25^1/2, or 5
Hyperboloid 2009-04-03 19:13:00
rrusczyk 2009-04-03 19:13:01
rrusczyk 2009-04-03 19:13:10
So what's our answer?
tinytim 2009-04-03 19:13:24
Adding these gives an answer of 469.
geoffreywong 2009-04-03 19:13:24
469
sj0201 2009-04-03 19:13:24
343+121+5 = 469
GoldenFrog1618 2009-04-03 19:13:24
469
Hyperboloid 2009-04-03 19:13:24
So the answer is 469.
sherlock 2009-04-03 19:13:24
469
rrusczyk 2009-04-03 19:13:29
Adding our results gives 343 + 121 + 5 = 469.
rrusczyk 2009-04-03 19:13:38
Two down. 13 to go.
rrusczyk 2009-04-03 19:13:43
mathq 2009-04-03 19:13:55
make a diagram
sj0201 2009-04-03 19:13:57
diagram!!
Recursive Acronym 2009-04-03 19:13:59
Draw a picture!
rrusczyk 2009-04-03 19:14:04
rrusczyk 2009-04-03 19:14:17
What are we likely to use to solve this problem?
dgreenb801 2009-04-03 19:14:24
similar triangles
tinytim 2009-04-03 19:14:24
Bunch of similar tirangles
Recursive Acronym 2009-04-03 19:14:24
So many similar right triangles...
rrusczyk 2009-04-03 19:14:30
We have lots of right triangles, and we have similar triangles all over the place.
rrusczyk 2009-04-03 19:14:47
We let AE = x so we can start building equations. (Notice that this is easier than letting AD = x, since we avoid fractions.)
rrusczyk 2009-04-03 19:14:55
What next?
all4math 2009-04-03 19:15:26
we use ratios
rrusczyk 2009-04-03 19:15:29
Which ones?
dgreenb801 2009-04-03 19:15:59
ABE and CBA are similar
dgreenb801 2009-04-03 19:15:59
AE/AB=AB/AC
MathWhiz2009 2009-04-03 19:15:59
ABE~ABC
m1sterzer0 2009-04-03 19:15:59
AB/AE=BC/AC
motoe123 2009-04-03 19:15:59
AE:AB=AB:BC
rrusczyk 2009-04-03 19:16:11
Here's a reason we focus on these:
Hyperboloid 2009-04-03 19:16:13
One involving AB, since we know its length.
MathWhiz2009 2009-04-03 19:16:22
actually ABE~BCA
rrusczyk 2009-04-03 19:16:29
Since AE = x, we have BC = 2x.
rrusczyk 2009-04-03 19:16:42
So what equation do we get?
MathWhiz2009 2009-04-03 19:16:58
x/100=100/(2x)
GoldenFrog1618 2009-04-03 19:16:58
2x^2=BA^2
sj0201 2009-04-03 19:16:58
2x/100 = 100/x
mathq 2009-04-03 19:16:58
x:100=100:2x
chessmaster7 2009-04-03 19:16:58
2x/100=100/x
rrusczyk 2009-04-03 19:17:01
From ABC ~ EAB, we have AE/AB = AB/BC, so x/100 = 100/(2x).
rrusczyk 2009-04-03 19:17:27
So?
tinytim 2009-04-03 19:17:53
x=50sqrt{2}
mismatchtea 2009-04-03 19:17:54
2x^2 = 10000
sj0201 2009-04-03 19:17:54
x^2 = 5000
GoldenFrog1618 2009-04-03 19:17:54
x=50*sqrt(2)
sa314 2009-04-03 19:17:54
x^2 = 5000
dgreenb801 2009-04-03 19:17:54
x=100/sqrt{2}=50sqrt{2}
mathspee 2009-04-03 19:17:54
10000=2x^2
rrusczyk 2009-04-03 19:17:57
rrusczyk 2009-04-03 19:18:03
Keep going . . .
rrusczyk 2009-04-03 19:18:09
What is AD?
Recursive Acronym 2009-04-03 19:18:24
Double.
tinytim 2009-04-03 19:18:24
so 2x is 100sqrt{2}
e^i_penguin 2009-04-03 19:18:24
100sqrt2
mathspee 2009-04-03 19:18:24
2x=100 root2
GoldenFrog1618 2009-04-03 19:18:24
AD=100*sqrt(2)
rrusczyk 2009-04-03 19:18:26
rrusczyk 2009-04-03 19:18:29
And what is our answer?
Yongyi781 2009-04-03 19:18:43
2x = 100sqrt(2) = 141
sj0201 2009-04-03 19:18:43
141
GoldenFrog1618 2009-04-03 19:18:43
141
chundai 2009-04-03 19:18:43
141.
mathspee 2009-04-03 19:18:43
141
tinytim 2009-04-03 19:18:43
Hullohihell0 2009-04-03 19:18:43
141
rrusczyk 2009-04-03 19:18:46
rrusczyk 2009-04-03 19:18:56
Three down.
rrusczyk 2009-04-03 19:18:59
rrusczyk 2009-04-03 19:19:34
Note: If you are here for the Intro Geometry class, please leave and then refresh the Classroom page. This is the AIME Math Jam.
rrusczyk 2009-04-03 19:19:40
We could just set this up like a typical arithmetic series problem. We haven + (n-2) + (n-4) + . . . + (n-2(m-1)) = 2009,where there are m children in the contest.
rrusczyk 2009-04-03 19:19:44
Ick.
rrusczyk 2009-04-03 19:19:50
Anyone have a nicer way to approach this?
rrusczyk 2009-04-03 19:20:29
What is typically an easier way to think about the sum of an arithmetic series?
Recursive Acronym 2009-04-03 19:21:01
First term plus last term divided by two times number of terms.
chundai 2009-04-03 19:21:01
the average of the first and last terms multiplied by the number of terms?
rrusczyk 2009-04-03 19:21:39
A nicer way of thinking about the sum of an arithmetic series is the average of the terms times the number of terms. Letting m be the number of students and t be the average, we now have simply mt = 2009.
rrusczyk 2009-04-03 19:21:46
It appears that m and t must divide 2009, but first we must make sure both are integers. Are they?
rrusczyk 2009-04-03 19:22:38
We have to make sure the *average*, t, is an integer.
Hyperboloid 2009-04-03 19:23:04
Yes
GoldenFrog1618 2009-04-03 19:23:04
n and k must have the same parity
nechesskid13 2009-04-03 19:23:04
The numbers all have the same parity, so the average is an integer.
rrusczyk 2009-04-03 19:23:07
Yes -- the average of the terms equals the average of the first and last terms. These two terms have the same parity (both odd or both even), so their average is an integer.
mismatchtea 2009-04-03 19:23:12
the prime factors of 2009 are 41 and 7 and 7
Hyperboloid 2009-04-03 19:23:12
rrusczyk 2009-04-03 19:23:18
rrusczyk 2009-04-03 19:23:22
We don't have many options for (m,t) to check. Do we have to try every factor of 2009 for m?
rrusczyk 2009-04-03 19:24:06
Are there any we know we'll be able to exclude?
mismatchtea 2009-04-03 19:24:09
only 3*2/2 possibilities
rrusczyk 2009-04-03 19:24:20
Why do we only have to test half the possibilities for m?
rrusczyk 2009-04-03 19:24:46
m and t are not the same -- they mean different things.
BenM 2009-04-03 19:25:41
t must be greater than m
Hyperboloid 2009-04-03 19:25:41
m<t
m1sterzer0 2009-04-03 19:25:41
we can't have anyone having to eat 'negative grapes' to make the average too low
tinytim 2009-04-03 19:25:41
The average must be greater than the number of people.
rrusczyk 2009-04-03 19:25:47
Students can't eat a negative number of grapes, so we only need to check those cases for which the last-place student eats a positive number of grapes.
tinytim 2009-04-03 19:26:09
Thus, we only need to try 1,7,and 41 for m.
rrusczyk 2009-04-03 19:26:14
Those cases are (m,t) = (1,2009); (7, 287); (41,49). If (m,t) = (49, 41), then we have 49 kids eating an average of 41 grapes, so the last place kid eats 41 - 2(24) = -7 grapes. Ooops! Similarly, if we have even more kids and a lower average of grapes, we'll have a kid eating a negative number of grapes, which would not be pretty.
rrusczyk 2009-04-03 19:26:34
What do we get for n in each of these cases?
tinytim 2009-04-03 19:27:31
rrusczyk 2009-04-03 19:27:36
So, all that's left is to find the smallest n. If (m,t) = (1,2009), we have n = 2009.
rrusczyk 2009-04-03 19:27:43
If (m,t) = (7,287), then 7 kids average 287 grapes, so n = 287 + 3*2 = 293.
rrusczyk 2009-04-03 19:27:48
If (m,t) = (41,49), then 41 kids average 49 grapes, and n = 49 + 2*20 = 89.
rrusczyk 2009-04-03 19:27:50
So. . . .
sucker0923 2009-04-03 19:28:03
smallest
Hullohihell0 2009-04-03 19:28:03
89 is the answer
sucker0923 2009-04-03 19:28:03
=89
lukezli 2009-04-03 19:28:03
89 is the answer.
rrusczyk 2009-04-03 19:28:14
The smallest of these is obviously 89, so that's our answer.
rrusczyk 2009-04-03 19:28:38
Four down in a half hour. Still on track to get that perfect 15!
rrusczyk 2009-04-03 19:28:43
rrusczyk 2009-04-03 19:28:57
rrusczyk 2009-04-03 19:29:12
Where do we start?
Hullohihell0 2009-04-03 19:29:24
where's the triangle?
rrusczyk 2009-04-03 19:29:42
The triangle is formed by connecting the outer points of tangency.
dgreenb801 2009-04-03 19:30:07
Canter of big circle, center of smaller circle, point of tangency collinear
Recursive Acronym 2009-04-03 19:30:07
Draw a lot of lines.
rrusczyk 2009-04-03 19:30:16
Let's play connect the dots.
m1sterzer0 2009-04-03 19:30:18
usually i start by drawing a lot of lines
rrusczyk 2009-04-03 19:30:30
What lines are the first ones to start with here?
tinytim 2009-04-03 19:30:56
AED(or AEC) is a good triangle to try and use.
chessmaster7 2009-04-03 19:30:57
AC, CD, AD, BA, BC, and BD
GoldenFrog1618 2009-04-03 19:30:57
CE, DE, BE
tinytim 2009-04-03 19:31:17
Connect centers of circles
rrusczyk 2009-04-03 19:31:21
rrusczyk 2009-04-03 19:31:29
We definitely want to start by connecting up the centers.
rrusczyk 2009-04-03 19:31:39
What's going to be our general strategy here?
chessmaster7 2009-04-03 19:32:25
now we can write in lengths using the given radiuses, and maybe try pythagoras or trig
tinytim 2009-04-03 19:32:25
Find expressions for lengths in terms of r, the radius of E.
rrusczyk 2009-04-03 19:32:41
Our goal is to build an equation for r, the radius of E.
rrusczyk 2009-04-03 19:33:04
So, let's find some lengths.
rrusczyk 2009-04-03 19:33:10
What lengths can we find?
geoffreywong 2009-04-03 19:33:43
AC
mismatchtea 2009-04-03 19:33:43
AC
rrusczyk 2009-04-03 19:33:46
What is AC?
tinytim 2009-04-03 19:34:17
10-2=8
geoffreywong 2009-04-03 19:34:17
8
mismatchtea 2009-04-03 19:34:17
10-2=8
chessmaster7 2009-04-03 19:34:17
8
daydreamer1116 2009-04-03 19:34:22
10-2?
rrusczyk 2009-04-03 19:34:48
We have AC = 10-2 = 8, since the radius of C is 2, and if we continue AC, it will hit the point of tangency of C and A.
daydreamer1116 2009-04-03 19:34:54
and AC=AD
rrusczyk 2009-04-03 19:34:58
True.
rrusczyk 2009-04-03 19:35:20
So, we have AC = 8. What other sides can we find (either find the length, or find them in terms of r).
mismatchtea 2009-04-03 19:35:26
AB = 10-3=7
daydreamer1116 2009-04-03 19:35:26
and doesn't AB=10-3=7?
rrusczyk 2009-04-03 19:35:37
Exactly. For the same reason, we have AB = 10 - 3 = 7.
GoldenFrog1618 2009-04-03 19:35:41
E is on line BA
rrusczyk 2009-04-03 19:35:51
True -- by symmetry, E is on AB.
MathWhiz2009 2009-04-03 19:35:54
but how does this help us
rrusczyk 2009-04-03 19:35:59
How does this help?
chessmaster7 2009-04-03 19:36:10
EC = r+2
rrusczyk 2009-04-03 19:36:21
EC = r + 2, since it is the sum of the radii of E and C.
rrusczyk 2009-04-03 19:36:37
(Please use capital letters for points - I don't read the ones that have lowercase letters for points.)
rrusczyk 2009-04-03 19:36:49
Are there any other lengths we can find?
tinytim 2009-04-03 19:37:04
m1sterzer0 2009-04-03 19:37:04
EA = EB-AB = (r+3)-7=r-4
GoldenFrog1618 2009-04-03 19:37:04
AE=r-4
rrusczyk 2009-04-03 19:37:41
We have AE = r-4, since the distance from A to the point of tangency of A and B is 7-3 = 4.
rrusczyk 2009-04-03 19:37:45
Now what?
tinytim 2009-04-03 19:38:16
Now we can use Law of Cosines
GoldenFrog1618 2009-04-03 19:38:17
EAD=60 degrees
dgreenb801 2009-04-03 19:38:17
EAD=60?
tinytim 2009-04-03 19:38:20
We have enough information to use Law of Cosines.
m1sterzer0 2009-04-03 19:38:20
we know angle EAD is 60 degrees
rrusczyk 2009-04-03 19:38:22
We have three sides of ACE: AC = 8, AE = r-4, CE = 2+r. . .
rrusczyk 2009-04-03 19:38:27
If we're stuck here, we ask ourselves what we haven't used yet. We haven't used the information about the equilateral triangle. The outer points of tangency on circle A are vertices of an equilateral triangle. The rays from A through B, C, and D will pass through these three points, so <BAC = <CAD = <DAB = 120. By symmetry, AE bisects <CAD, so <CAE = 60. Now, we're set.
rrusczyk 2009-04-03 19:38:38
In triangle ACE, we have AC = 8, CE = 2 + r, AE = r - 4, and <CAE = 60.
rrusczyk 2009-04-03 19:39:22
Now we break out the law of cosines.
chessmaster7 2009-04-03 19:39:33
(r+2)^2=8^2+(r-4)^2-2(8)(r-4)*1/2
tinytim 2009-04-03 19:39:33
rrusczyk 2009-04-03 19:39:38
rrusczyk 2009-04-03 19:39:46
And what do we find?
chessmaster7 2009-04-03 19:40:38
we get 20r=108
m1sterzer0 2009-04-03 19:40:38
r=108/20=27/5
charles.du 2009-04-03 19:40:38
27/5 so answer is 32
rrusczyk 2009-04-03 19:40:43
tinytim 2009-04-03 19:41:14
or we could have Pythagoras bashed...
rrusczyk 2009-04-03 19:41:25
You can indeed build some right triangles and bash away.
charles.du 2009-04-03 19:41:30
yeah, extend BA to meet CD
rrusczyk 2009-04-03 19:41:39
Like that. You can work out that solution on your own.
rrusczyk 2009-04-03 19:41:45
For now, we're on to #6!!
HiDN428 2009-04-03 19:41:55
we would get 32 using the pythag as well
rrusczyk 2009-04-03 19:41:59
I sure hope so :)
rrusczyk 2009-04-03 19:42:00
rrusczyk 2009-04-03 19:42:10
"At least" in a counting problem. What should that make us think?
jeffreyyan8 2009-04-03 19:42:21
well right off the bat, i used PIE-principle of inclusion exclusion
rrusczyk 2009-04-03 19:42:32
That's one thing I sometimes think, but usually I think this first:
BenM 2009-04-03 19:42:42
find the opposite
spelljack825 2009-04-03 19:42:42
complement counting
chundai 2009-04-03 19:42:42
complimentary counting!
tinytim 2009-04-03 19:42:42
Count the total number of 5-element subsets and then use complementary counting.
GoldenFrog1618 2009-04-03 19:42:42
complimentary counting
maxschindler 2009-04-03 19:42:42
complementary counting
rrusczyk 2009-04-03 19:42:49
Any time we see "at least" in a counting problem, we should consider counting what we don't want instead of directly counting what we want. (We at AoPS sometimes call this "complementary counting".)
rrusczyk 2009-04-03 19:43:04
Complimentary counting is nice, but it's complementary counting that we want to do here.
GoldenFrog1618 2009-04-03 19:43:17
without restriction there are C(14,5)
tinytim 2009-04-03 19:43:17
The total number of sets is easy. Simply 14C5=2002.
rrusczyk 2009-04-03 19:43:23
rrusczyk 2009-04-03 19:43:39
Next, we have to count the number of five-element subsets that don't satisfy the problem. In words, what are these?
tinytim 2009-04-03 19:44:11
Now we think of how to "construct" a set such no two members are consecutive.
e714212835 2009-04-03 19:44:11
no two numbers are consecutive
GoldenFrog1618 2009-04-03 19:44:11
no two numbers are consecutive
Hullohihell0 2009-04-03 19:44:11
ones where no two numbers are consecutive
happysunnyshine 2009-04-03 19:44:11
no consecutive numbers
rrusczyk 2009-04-03 19:44:15
These are the five-element subsets such that no two numbers are consecutive.
rrusczyk 2009-04-03 19:44:21
How many of these are there?
Hyperboloid 2009-04-03 19:45:05
10C5 = 252
BenM 2009-04-03 19:45:05
10 chose 5
lameneses 2009-04-03 19:45:05
rrusczyk 2009-04-03 19:45:15
qwang 2009-04-03 19:45:32
If we don't want any 5 numbers to be consecutive, consider 5 placemarkers that we place numbers around. for every arrangement of numbers around the placeholders, there's one set.
dgreenb801 2009-04-03 19:45:32
We draw a bijection: for each possibility, we can take out a number in between each of the selected numbers
BenM 2009-04-03 19:45:32
because you insert 4 spaces between the numbers and 14-4=10
rrusczyk 2009-04-03 19:45:44
I'm going to do a quick counting lesson to go through what these answers mean.
rrusczyk 2009-04-03 19:45:49
There are a number of ways to see this.
rrusczyk 2009-04-03 19:45:59
We can view the first n integers as n chairs in a row. So, we have 14 chairs in a row, and we must pick 5 chairs such that no two chosen chairs are adjacent. Here's an example:
rrusczyk 2009-04-03 19:46:03
OXXOXXXOXOXXOX
rrusczyk 2009-04-03 19:46:07
The O's are the chosen chairs.
tinytim 2009-04-03 19:46:11
Since no two are consective we can "take out" one number between each of the 5 numbers.
rrusczyk 2009-04-03 19:46:15
If we take away one un-chosen chair between each successive pair of O's, we have
rrusczyk 2009-04-03 19:46:19
OXOXXOOXOX
rrusczyk 2009-04-03 19:46:33
We have 10 chairs now, and 5 of them are chosen. What happens when we reverse this process, and jam an extra X between each successive pair of O's?
spelljack825 2009-04-03 19:47:12
you get the original
tinytim 2009-04-03 19:47:13
We get a set that has no two consecutive O's
rrusczyk 2009-04-03 19:47:15
We get back the original arrangement of 14 chairs:
rrusczyk 2009-04-03 19:47:20
OXXOXXXOXOXXOX
rrusczyk 2009-04-03 19:47:25
How does this help us count the number of ways we can choose 5 of the 14 chairs such that no two are adjacent?
tinytim 2009-04-03 19:47:49
So this corresponds o arranging 5 x's and 5 o's which is 10C5.
spelljack825 2009-04-03 19:47:49
the number of combinations is equivalent
rrusczyk 2009-04-03 19:48:02
For any such arrangement, we can take out 4 X's, one from between each successive pair of O's (which we can always do because no two chairs are adjacent!). We then get a row of 5 X's and 5 O's. We can reverse the process. For any row of 5 X's and 5 O's, we can add an X between each successive pair of O's and produce a row with 9 X's and 5 O's such that no two O's are adjacent (since we added an X between each successive pair).
rrusczyk 2009-04-03 19:48:16
Since each 14-letter string of 9 X's and 5 O's and no O's adjacent can be turned into a single 10-letter string of 5 X's and 5 O's, and vice-versa, we say that we have a 1-to-1 correspondence between the two sets of strings.
chessmaster7 2009-04-03 19:48:23
so there are 10C5 possbilities on ways to be able to removes X's without removing O's, or in other words, having chairs that are not adjacent
rrusczyk 2009-04-03 19:48:30
Therefore, the number of 5-element subsets of the first 14 integers with no two consecutive integers equals the number of 5-element subsets of the first 10 integers.
rrusczyk 2009-04-03 19:48:42
So, how do we finish?
Hyperboloid 2009-04-03 19:48:49
charles.du 2009-04-03 19:48:50
you just do 2002-252=1750
tinytim 2009-04-03 19:48:50
And the answer is 14C5-10C5=2002-252=1750=750 (mod 1000)
BenM 2009-04-03 19:48:50
so the answer is 14C5-10C5
Hyperboloid 2009-04-03 19:48:54
Subtract the complement form the total.
rrusczyk 2009-04-03 19:48:59
rrusczyk 2009-04-03 19:49:22
And that takes care of #6!!
rrusczyk 2009-04-03 19:49:32
Speaking of !!, it's time for #7!!
rrusczyk 2009-04-03 19:49:36
rrusczyk 2009-04-03 19:49:54
Icky notation. Whenever I see that on problem 6-10 on the AIME, my first thought is, "I'll bet this problem is way easier than it looks."
rrusczyk 2009-04-03 19:50:01
Let's see if I'm right. Where should we start?
all4math 2009-04-03 19:50:09
what does that where e thing mean
rrusczyk 2009-04-03 19:50:13
It's a summation.
rrusczyk 2009-04-03 19:50:33
It means you put in i=1, then i=2, then i=3, all the way up to 2009, and add up all the results you get.
rrusczyk 2009-04-03 19:50:50
What did you do when you first saw the problem?
rrusczyk 2009-04-03 19:51:01
Did you immediately start writing general formulas?
Hullohihell0 2009-04-03 19:51:04
panic
rrusczyk 2009-04-03 19:51:24
Gotta get over that! I used to, too, but then I realized the nasty notation problems are often easiest!
chessmaster7 2009-04-03 19:51:28
calculated the first few n's
federernadal 2009-04-03 19:51:29
look for a pattern
m1sterzer0 2009-04-03 19:51:29
started trying the first few terms to see if I saw a pattern
BenM 2009-04-03 19:51:29
write a few terms
happysunnyshine 2009-04-03 19:51:29
try a few
rrusczyk 2009-04-03 19:51:33
This is what I usually do.
rrusczyk 2009-04-03 19:51:41
Let's look at the first few terms, if only to get used to the notation. What is the first term?
tinytim 2009-04-03 19:51:54
1/2
nechesskid13 2009-04-03 19:51:54
1/2
rrusczyk 2009-04-03 19:51:58
rrusczyk 2009-04-03 19:52:00
Not too interesting. Next term?
BenM 2009-04-03 19:52:13
3/8
tinytim 2009-04-03 19:52:13
3/8
mathq 2009-04-03 19:52:13
3/8
jweisblat14 2009-04-03 19:52:13
3/8
rrusczyk 2009-04-03 19:52:16
rrusczyk 2009-04-03 19:52:17
Next?
rrusczyk 2009-04-03 19:52:43
Simplify your result!
GoldenFrog1618 2009-04-03 19:52:47
5/16
mathq 2009-04-03 19:52:47
5/16
jweisblat14 2009-04-03 19:52:47
5/16
nechesskid13 2009-04-03 19:52:47
15/48 = 5/16
rrusczyk 2009-04-03 19:52:50
rrusczyk 2009-04-03 19:53:06
Next?
geoffreywong 2009-04-03 19:53:45
35/128
jweisblat14 2009-04-03 19:53:45
35/128
e714212835 2009-04-03 19:53:45
35/128
late20s 2009-04-03 19:53:45
35/128
rrusczyk 2009-04-03 19:53:47
rrusczyk 2009-04-03 19:53:52
Are we ready to make any conjectures yet?
charles.du 2009-04-03 19:54:05
all the numerators are odd, and all the denominators are even
mismatchtea 2009-04-03 19:54:06
so the top appears to be consecutive odds, while the bottom is 2 raised to the nth power
GoldenFrog1618 2009-04-03 19:54:06
denominator is always a power of 2
GoldenFrog1618 2009-04-03 19:54:16
do you mean i=4 not 7?
rrusczyk 2009-04-03 19:54:25
Yes, I meant i=4 for that last case.
rrusczyk 2009-04-03 19:54:43
And i=2, i=3 for the previous 2.
tinytim 2009-04-03 19:54:48
if the denominator is always a power of 2, then b=1
rrusczyk 2009-04-03 19:54:51
It sure looks like the denominator will always be a power of 2. Will the next one be a power of 2?
rrusczyk 2009-04-03 19:55:18
How can we easily see what the next one is?
jweisblat14 2009-04-03 19:55:20
10 has a five
rrusczyk 2009-04-03 19:55:28
Yes it does, and what happens to that five?
spelljack825 2009-04-03 19:55:33
yes, 5|10
charles.du 2009-04-03 19:55:33
but it cancels
e714212835 2009-04-03 19:55:33
we multiply the denomonator by 10, and the 5 gets cancelled
rrusczyk 2009-04-03 19:55:37
Yes. The next term is 9/10 the previous term. The 5 in the 10 cancels with the 5 in the numerator.
rrusczyk 2009-04-03 19:55:42
Will this continue forever?
GoldenFrog1618 2009-04-03 19:55:54
yes
geoffreywong 2009-04-03 19:55:54
yes
jweisblat14 2009-04-03 19:55:54
yes
nechesskid13 2009-04-03 19:55:54
The numerator will simplify all odd factors in the denominator.
tinytim 2009-04-03 19:55:54
Yes.
qwang 2009-04-03 19:56:18
the only way the denominator isn't a power of two is if it has 2*odd somewhere in there. however, the 'odd' will always cancel with one on top.
rrusczyk 2009-04-03 19:56:28
A true observation and .. . .
tinytim 2009-04-03 19:56:30
Because this is the AIME we have no need to prove it.:-D
rrusczyk 2009-04-03 19:56:42
If this were the USAMO, qwang's statement would need to be proved.
rrusczyk 2009-04-03 19:57:01
It's not enough to say that for every 2n in the denom, there's an n in the numerator. What happens if n is even?
rrusczyk 2009-04-03 19:57:10
We can get a sense for why it is true like this:
rrusczyk 2009-04-03 19:57:16
rrusczyk 2009-04-03 19:57:20
For each odd integer k greater than 1, we have:
rrusczyk 2009-04-03 19:57:23
k appears in the numerator before 2k appears in the denominator (since the denominator of the term with k in the numerator is just k+1).
rrusczyk 2009-04-03 19:57:27
3k appears in the numerator before 4k appears in the denominator.
rrusczyk 2009-04-03 19:57:30
5k appears in the numerator before 6k appears in the denominator
rrusczyk 2009-04-03 19:57:34
And so on.
rrusczyk 2009-04-03 19:57:38
This isn't perfectly rigorous, since more factors of k will appear in the coefficients of k in our argument. But then we can apply the same argument to those coefficients to take care of those factors, and so on.
rrusczyk 2009-04-03 19:58:01
I mention this argument because it is *exactly* the kind of convincing argument some of you will write on the USAMO, thinking you have a proof.
rrusczyk 2009-04-03 19:58:10
And then you'll get a 0/7 and wonder what happened.
rrusczyk 2009-04-03 19:58:47
Everything I wrote above is true, but the "and so on" part is not rigorous. We would have to prove that the "and so on" does continue forever.
charles.du 2009-04-03 19:58:49
wait so what gets you partial credit on USAMO
rrusczyk 2009-04-03 19:59:03
It's very hard to get partial credit on the USAMO -- ask about it on the message board.
rrusczyk 2009-04-03 19:59:14
Does anyone have a more rigorous explanation for why the odds always cancel out? (At this point, the observation above is enough to finish the problem, but let's really nail this one down by proving we're right for sure.)
GoldenFrog1618 2009-04-03 20:01:22
2^i*i!
tinytim 2009-04-03 20:01:23
Start by noting (2i)!!=2^i(i!)... Is that still a typo...?
rrusczyk 2009-04-03 20:01:47
We have (2i)!! = 2^i * i!
rrusczyk 2009-04-03 20:02:05
Can we relate (2i)!! and (2i-1)! to something that we are more comfortable with?
rrusczyk 2009-04-03 20:02:25
Sorry, that should be (2i)!! and (2i-1)!!
spelljack825 2009-04-03 20:02:27
factorials!
rrusczyk 2009-04-03 20:02:37
And how can we relate these to factorials?
spelljack825 2009-04-03 20:03:10
(2i)!! * (2i-1)!! = 2i!
rrusczyk 2009-04-03 20:03:29
rrusczyk 2009-04-03 20:03:55
Where have you seen (2m)! and m! together before?
tinytim 2009-04-03 20:04:20
2mCm!
chessmaster7 2009-04-03 20:04:20
combinations?
rrusczyk 2009-04-03 20:04:24
rrusczyk 2009-04-03 20:04:45
rrusczyk 2009-04-03 20:04:47
How else can we write that?
rrusczyk 2009-04-03 20:04:56
(Using our !! notation.)
rrusczyk 2009-04-03 20:05:37
(Also, please write (2m)! rather than 2m!. There's a big difference!)
chundai 2009-04-03 20:05:39
why is (2i)!!*(2i-1)!!=2i!
rrusczyk 2009-04-03 20:06:07
The (2i)!! has all the evens and the (2i-1)!! has all the odds. Together, they have all the numbers from 1 to 2i. That's (2i)!
rrusczyk 2009-04-03 20:06:31
Just above, we saw a way to write (2m)! in terms of !! -- what was that?
happysunnyshine 2009-04-03 20:07:00
2m!!*(2m-1)!!
rrusczyk 2009-04-03 20:07:11
rrusczyk 2009-04-03 20:07:26
OK. Keep going. We saw a way to bring m! into the mix, too. What was that?
e714212835 2009-04-03 20:08:01
(2^m)(m!) = (2m)!!
rrusczyk 2009-04-03 20:08:03
We had (2m)!! = 2^m * m!.
rrusczyk 2009-04-03 20:08:07
So...
GoldenFrog1618 2009-04-03 20:08:08
2^m(2m-1)!!/m!
rrusczyk 2009-04-03 20:08:13
rrusczyk 2009-04-03 20:08:17
How does this help us?
rrusczyk 2009-04-03 20:09:06
What do we know is true about this number?
late20s 2009-04-03 20:09:08
a integer
rrusczyk 2009-04-03 20:09:24
This number is an integer because all C(n,r) are integers. Why does that help?
happysunnyshine 2009-04-03 20:09:26
why is (2^m)(m!) = (2m)!!?
rrusczyk 2009-04-03 20:10:00
(2m)!! = 2m * (2m-2) * . . . * 1 = 2^m[(m)*(m-1) *(m-2)* . . . *1]
rrusczyk 2009-04-03 20:10:41
How does the fact that this number is an integer help us? What does it tell us about (2m-1)!! and m!?
geoffreywong 2009-04-03 20:11:24
they cancel out all odd numbers for m!
charles.du 2009-04-03 20:11:44
we know m!|2^m(2m-1)!!
rrusczyk 2009-04-03 20:11:55
Exactly. Because 2^m(2m-1)!! / m! is an integer, all the odd factors of m! cancel with (2m-1)!!.
rrusczyk 2009-04-03 20:12:16
Going back to (2m-1)!! / (2m)!!, what do we now know?
tinytim 2009-04-03 20:13:04
Which leaves us with just powers of 2
tinytim 2009-04-03 20:13:04
that just equals (random stuff)/(2^i)
GoldenFrog1618 2009-04-03 20:13:04
the odd factors cancel
rrusczyk 2009-04-03 20:13:16
rrusczyk 2009-04-03 20:13:23
We now know that all the denominators are powers of 2, so all we have to do is find the highest one.
rrusczyk 2009-04-03 20:13:48
That's the 7/7 proof. Didn't need it for the AIME, but now you should remember that "and so on" won't get you far on the USAMO next month.
rrusczyk 2009-04-03 20:14:01
rrusczyk 2009-04-03 20:14:05
This is a problem we know how to handle. How do we do it?
geoffreywong 2009-04-03 20:14:39
pull out the 2^i first
rrusczyk 2009-04-03 20:14:47
rrusczyk 2009-04-03 20:14:50
And how do we do that?
GoldenFrog1618 2009-04-03 20:15:21
[2009/2]+[2009/4]+[2009/8]+...
kevmo314 2009-04-03 20:15:21
Count those divisible by 2, then 4, then 8, and so on, and add them up.
Hullohihell0 2009-04-03 20:15:21
count 2s then 4s then 8s and so on
spelljack825 2009-04-03 20:15:27
[2009/2] + [2009/2^2] +...
chessmaster7 2009-04-03 20:15:30
count factors of largest factor of 2 smaller than 2009, then 2nd largest factor, 3rd largest, and so on
rrusczyk 2009-04-03 20:15:33
rrusczyk 2009-04-03 20:15:55
So. . .
tinytim 2009-04-03 20:16:11
add 2009 to that
rrusczyk 2009-04-03 20:16:20
tinytim 2009-04-03 20:16:53
Which gives 2^{4010} in the denominator. So a/10=401
chessmaster7 2009-04-03 20:16:53
so a=4010 from the original question
jweisblat14 2009-04-03 20:16:53
4001*1/10=401
rrusczyk 2009-04-03 20:16:56
charles.du 2009-04-03 20:16:59
I just went ahead and found a for this problem to be 401 and realized that b had to be one, or ab/10 would be over 1000 :D
rrusczyk 2009-04-03 20:17:08
Funny -- that's a bit of a bug in the problem :)
rrusczyk 2009-04-03 20:17:34
(This would have been a better USAMTS problem than an AIME problem -- you can get the answer without doing the most interesting part of the problem.)
rrusczyk 2009-04-03 20:17:42
But now we're ready for number 8!
rrusczyk 2009-04-03 20:17:48
rrusczyk 2009-04-03 20:17:54
One approach is to set up a big infinite series indexed by the number of rolls it takes each person to roll a 6.
rrusczyk 2009-04-03 20:18:00
That seems ugly to me.
rrusczyk 2009-04-03 20:18:11
What's another way to approach the problem?
rrusczyk 2009-04-03 20:18:48
I see some pretty complicated suggestions!
nechesskid13 2009-04-03 20:19:02
Geometric series
rrusczyk 2009-04-03 20:19:18
That will work, but let's see if we can find a simpler approach.
rrusczyk 2009-04-03 20:19:22
What can happen on the first roll?
m1sterzer0 2009-04-03 20:19:29
recognize that if neither person rolls a 6 on the first roll, you are back to where you started
rrusczyk 2009-04-03 20:19:35
Very interesting observation!
rrusczyk 2009-04-03 20:19:38
Let's investigate.
rrusczyk 2009-04-03 20:19:54
So, what are the things that could happen on the first roll?
charles.du 2009-04-03 20:19:58
6 or not 6 for either person
tinytim 2009-04-03 20:20:05
6 or no 6
Hullohihell0 2009-04-03 20:20:05
6 or not a 6
rrusczyk 2009-04-03 20:20:14
If they both get a 6 right way, they win.
jweisblat14 2009-04-03 20:20:19
1/36 for both 6
rrusczyk 2009-04-03 20:20:24
This occurs with probability 1/36.
rrusczyk 2009-04-03 20:20:33
What about neither get 6?
tinytim 2009-04-03 20:20:46
25/36
nechesskid13 2009-04-03 20:20:46
25/36
late20s 2009-04-03 20:20:46
25/36 for reatarting the game
Hullohihell0 2009-04-03 20:20:46
25/36
charles.du 2009-04-03 20:20:46
25/36
rrusczyk 2009-04-03 20:20:48
If they both don't get a 6, they'll have to roll again.
rrusczyk 2009-04-03 20:20:52
This occurs with probability 25/36.
geoffreywong 2009-04-03 20:20:56
goes back to original setup
rrusczyk 2009-04-03 20:21:02
And then we're back where we started.
rrusczyk 2009-04-03 20:21:06
So the rest of the time, with probability 10/36, exactly one of them will roll a 6.
rrusczyk 2009-04-03 20:21:09
Then what happens?
nechesskid13 2009-04-03 20:21:24
The other has to roll a six
Hullohihell0 2009-04-03 20:21:24
the other has to roll
deltaren 2009-04-03 20:21:24
the other has to roll a 6
rrusczyk 2009-04-03 20:21:27
They win if the other person rolls a 6 on their next roll.
chessmaster7 2009-04-03 20:21:33
then 10/36*1/6=10/216=5/108 for then to win
rrusczyk 2009-04-03 20:21:54
This occurs with probability 1/6, so the probability is (10/36)(1/6) that they'll win this way.
rrusczyk 2009-04-03 20:22:09
How can we use all these observations *to finish the problem*.
tinytim 2009-04-03 20:22:37
Let x be the answer. x=1/36+10/36*1/6+25/36x
sa314 2009-04-03 20:22:37
write an equation
GoldenFrog1618 2009-04-03 20:22:37
5/108+1/6+25/36p=p
rrusczyk 2009-04-03 20:22:43
Exactly.
rrusczyk 2009-04-03 20:22:49
We let p be the probability they win.
rrusczyk 2009-04-03 20:23:21
We have p = (win on first roll) + (win on one rolling a 6 first and the other rolling a 6 second) + (win on neither rolling a 6 first).
rrusczyk 2009-04-03 20:23:25
In other words:
rrusczyk 2009-04-03 20:23:32
rrusczyk 2009-04-03 20:23:40
The first term is "win on first roll"
rrusczyk 2009-04-03 20:23:58
The second term is when one rolls a 6 on the first turn and the other rolls a 6 on the next turn.
rrusczyk 2009-04-03 20:24:10
Otherwise, with probability 25/36, they get to try again and win with probability p --- that's the third term.
rrusczyk 2009-04-03 20:24:39
We add up the probability of winning in each case, and we get the overall probability.
rrusczyk 2009-04-03 20:24:46
So, we have that equation!
rrusczyk 2009-04-03 20:24:52
Now we just have to solve for p.
chessmaster7 2009-04-03 20:25:25
we get 11p/36=8/108=4/54=2/27, so 72=297p and p=24/99=8/33
tinytim 2009-04-03 20:25:25
11/36p=8/108 gives p=8/33
rrusczyk 2009-04-03 20:25:32
Now it's just an algebra problem.
rrusczyk 2009-04-03 20:25:38
Multiply through by 36 and collect terms:
rrusczyk 2009-04-03 20:25:43
rrusczyk 2009-04-03 20:25:46
rrusczyk 2009-04-03 20:25:48
Thus the answer is 8 + 33 = 041.
mismatchtea 2009-04-03 20:25:51
wow, nice....just like solving an infinitely continuing fraction
rrusczyk 2009-04-03 20:26:16
We go through problems like these in our Intermediate Counting book.
m1sterzer0 2009-04-03 20:26:21
not bad --- beats pounding through the geometric series
ThinkFlow 2009-04-03 20:26:21
So... when the problem can return to a previous state, we can usually do something like this?
rrusczyk 2009-04-03 20:26:39
Exactly -- we call the strategy we use for problems like this "state diagrams".
asdfjklasdfjkl 2009-04-03 20:26:40
im buying that!
rrusczyk 2009-04-03 20:26:43
:)
rrusczyk 2009-04-03 20:26:50
And now for number 9
rrusczyk 2009-04-03 20:26:58
spelljack825 2009-04-03 20:27:25
bijection!
dgreenb801 2009-04-03 20:27:25
relate the two
rrusczyk 2009-04-03 20:27:34
How can we relate the two?
Hyperboloid 2009-04-03 20:27:40
What is a bijection?
tinytim 2009-04-03 20:27:40
1-1 correspondence
rrusczyk 2009-04-03 20:27:55
A bijection is a 1-1 correspondence, like we did in an earlier problem.
asdfjklasdfjkl 2009-04-03 20:27:57
notice that when y=3 or more its the same problem
Wikipedian1337 2009-04-03 20:27:58
In the second equation, if y > 3, there it's the same as the first equation.
ThinkFlow 2009-04-03 20:28:33
why?
rrusczyk 2009-04-03 20:28:38
Good question -- why?
dgreenb801 2009-04-03 20:29:10
For each solution x,y,z in the second equation, there exists a solution x,y+3,z in the first
charles.du 2009-04-03 20:29:10
because if (x,y,z) solves the second one, (x,y+3,z) solves the first one
chessmaster7 2009-04-03 20:29:12
3*3 (which is y) yields 9, which can be added to 2000 to get 2009. but since y is not fixed, the equations become the same again
rrusczyk 2009-04-03 20:29:35
We can pair up solutions:(x,y+3,z) to the first <---> (x,y,z) to the second for y > 0.
tinytim 2009-04-03 20:29:52
For EVERY soltuion to the second equation, we can add 3 to the y value to get a soltuion to the first eqaution. Thus we count the number of soltuions to the first equation that we CANT get back to the first equation by subtracting 3 from the y value.
rrusczyk 2009-04-03 20:29:54
So we have a 1-1 correspondence of solutions:(x,y',z) to the first with y' > 3 <----> (x,y,z) to the secondwhere y'= y+3.
rrusczyk 2009-04-03 20:30:00
This pairs every solution of the second equation to a solution of the first equation with y>3, and vice versa.
rrusczyk 2009-04-03 20:30:09
So what are the "left over" solutions to the first equation?
mathq 2009-04-03 20:30:22
so we solve for y=1,2,3
ThinkFlow 2009-04-03 20:30:22
y<=3
BenM 2009-04-03 20:30:22
y=1,2,3
rrusczyk 2009-04-03 20:30:36
We have the solutions where y is 1, 2 or 3.
rrusczyk 2009-04-03 20:30:57
(We have to include the y=3 case because we are interested in *positive* integer solutions, so we can't have y=0 in the second equation.)
charles.du 2009-04-03 20:31:03
and y cant be 2 because then the whole LHS would be even, and 2009 is odd
funtwo 2009-04-03 20:31:03
y cant be 2 because of parity
rrusczyk 2009-04-03 20:31:07
Noting that y must be odd, we see we must have y=1 or y=3.
rrusczyk 2009-04-03 20:31:15
So, now we have a much simpler problem.
tinytim 2009-04-03 20:31:26
If y=1 then we must find the number of solutions to 2(x+2z)=2009-3 or x+2z=1003
rrusczyk 2009-04-03 20:31:49
rrusczyk 2009-04-03 20:32:22
How do we count the solutions to 4x + 2z = 2006? (We can divide by 2 to get 2x + z = 1003.)
BenM 2009-04-03 20:32:55
x can range from 1 to 501 and there is a unique z for each x
mathq 2009-04-03 20:32:55
total 501 solutions
FantasyLover 2009-04-03 20:32:55
z=1, 3, ..., 1001
charles.du 2009-04-03 20:33:03
everything from (501,1) to (1,1001) works, so we have 501 solutions
rrusczyk 2009-04-03 20:33:05
We have z = 1003-2x, so x can be anything from 1 to 501.
rrusczyk 2009-04-03 20:33:19
And for 4x + 2z = 2000 (otherwise known as 2x + z = 1000)?
tinytim 2009-04-03 20:33:51
Similarily for 2x+z=1000, x can be anything from 1 to 499.
funtwo 2009-04-03 20:33:51
x=1-499 because z must be positive
BenM 2009-04-03 20:33:51
x ranges from 1 to 499
e714212835 2009-04-03 20:33:51
z=1000 - 2x, so x can be anything from 1 to 499
mathq 2009-04-03 20:33:51
(1,998) to (499,2), 499 solutions
rrusczyk 2009-04-03 20:33:53
We have z = 1000 - 2x, so x can be anything from 1 to 499.
rrusczyk 2009-04-03 20:34:05
So /. . . .
jweisblat14 2009-04-03 20:34:09
1000
mathq 2009-04-03 20:34:09
501+499=1000
charles.du 2009-04-03 20:34:11
our answer is 499+501/1000 = OMG 000
mathq 2009-04-03 20:34:11
sothe answer is 0
rrusczyk 2009-04-03 20:34:14
So we have a total of 501 + 499 = 1000 solutions.
tinytim 2009-04-03 20:34:17
Adding the number of solutions to the two equations gives 1000. THis means that m-n=1000 and the answer is 000!:-o
BenM 2009-04-03 20:34:27
000 mod 1000
jweisblat14 2009-04-03 20:34:27
answer = 000
spelljack825 2009-04-03 20:34:27
499 + 501 = 0 (mod 1000) :-c
rrusczyk 2009-04-03 20:34:28
The answer is thus 000.
mathq 2009-04-03 20:34:43
rare to get 0 for the answer...
rrusczyk 2009-04-03 20:34:58
It happens more than average. It has happened twice.
Smartguy 2009-04-03 20:35:20
wasn't there one in 2000?
tinytim 2009-04-03 20:35:20
Once on the 2000 AIME i believe
rrusczyk 2009-04-03 20:35:24
One more problem, and then I'm going to take a short break.
rrusczyk 2009-04-03 20:35:29
rrusczyk 2009-04-03 20:35:52
Not exactly an appealing problem with all those words.
geoffreywong 2009-04-03 20:35:55
diagram?
e^i_penguin 2009-04-03 20:35:55
draw a picture
Hullohihell0 2009-04-03 20:35:55
draw a diagram
rrusczyk 2009-04-03 20:36:01
A lot of words for a pretty simple diagram. Of course, I drew it wrong the first time I saw the problem.
rrusczyk 2009-04-03 20:36:04
In the interests of not being here all night, here's the correct diagram:
rrusczyk 2009-04-03 20:36:09
mathq 2009-04-03 20:36:12
ABC makes an right triangle
rrusczyk 2009-04-03 20:36:24
We could do this with analytic geometry and/or bash it out with trig, but let's see if we can find a nicer solution without those crutches.
rrusczyk 2009-04-03 20:36:32
We know that AB = 5, BC = 12, AC = 13, and the angles are marked as in the diagram. What else can we determine?
tinytim 2009-04-03 20:37:21
Extend DC so it meets line AB at sy E.
charles.du 2009-04-03 20:37:21
extend CD to AB to get a nice isosceles triangle
wangsacl 2009-04-03 20:37:21
I prefer to intersect the extension of both AB and CD
rrusczyk 2009-04-03 20:38:33
Interesting observation -- write me a full solution with that and I'll post it at the end :)
rrusczyk 2009-04-03 20:38:43
I see many of you suggesting this:
GoldenFrog1618 2009-04-03 20:38:48
angle bisector
Hullohihell0 2009-04-03 20:38:48
well you could use angle bisector theorem possibly
Wikipedian1337 2009-04-03 20:38:48
angle bisector theorem
rrusczyk 2009-04-03 20:38:58
We have an angle bisector of an angle in triangle ABC.
rrusczyk 2009-04-03 20:39:02
rrusczyk 2009-04-03 20:39:10
Applying the Angle Bisector Theorem to angle bisector AF in triangle ABC, we have BF/CF = AB/AC. So, what are BF and CF?
charles.du 2009-04-03 20:39:50
10/3 and 26/3
chessmaster7 2009-04-03 20:39:56
10/3 = BF and 26/3 = CF
rrusczyk 2009-04-03 20:40:04
Since AB is 5/18 of AB+AC, we know that BF is 5/18 of BF + FC, which is 12. Therefore, BF = 12*(5/18) = 10/3, and FC = 12 - BF= 26/3.
rrusczyk 2009-04-03 20:40:18
Now what might we be on the lookout for?
ThinkFlow 2009-04-03 20:40:33
similar triangles
tinytim 2009-04-03 20:40:33
Similar Tiangles
chessmaster7 2009-04-03 20:40:38
similar triangles?
rrusczyk 2009-04-03 20:40:40
We have a right angle. We have equal angles. We should look for similar triangles. Do we have any?
chessmaster7 2009-04-03 20:41:07
not yet...
rrusczyk 2009-04-03 20:41:11
That's the spirit.
rrusczyk 2009-04-03 20:41:15
not *yet*
billy314 2009-04-03 20:41:20
we could draw some auxillary lines
happysunnyshine 2009-04-03 20:41:21
No, but we can create
ThinkFlow 2009-04-03 20:41:26
Draw some
rrusczyk 2009-04-03 20:41:30
How can we make some easily?
rrusczyk 2009-04-03 20:42:06
(I think there are a lot of ways to finish from here.)
billy314 2009-04-03 20:42:09
Draw the segment from D parallel to segment BA
ThinkFlow 2009-04-03 20:42:10
Draw D perpendicular to BC
rrusczyk 2009-04-03 20:42:13
We draw the altitude from D to BC.
rrusczyk 2009-04-03 20:42:17
rrusczyk 2009-04-03 20:42:26
Now what?
tinytim 2009-04-03 20:42:45
ABF is similar to DHF
chessmaster7 2009-04-03 20:42:45
find DF and FA using similar triangles and we have answer!
funtwo 2009-04-03 20:42:45
BAF~HDF
ThinkFlow 2009-04-03 20:42:50
We find the ratio of similarity, and we can find via pythag AF, so we find our answer
rrusczyk 2009-04-03 20:42:57
We now have a problem we know how to handle.
rrusczyk 2009-04-03 20:43:02
Let's knock off the details.
rrusczyk 2009-04-03 20:43:05
We have DHF ~ ABF.
rrusczyk 2009-04-03 20:43:07
What else?
rrusczyk 2009-04-03 20:43:15
(Any other similarities?)
chessmaster7 2009-04-03 20:43:28
CHD and CBA
ThinkFlow 2009-04-03 20:43:28
DHC similar to ABC
funtwo 2009-04-03 20:43:28
CDH and CAB
rrusczyk 2009-04-03 20:43:30
We also have DHC ~ ABC.
rrusczyk 2009-04-03 20:43:50
OK, let's get to work with these. How? What's our first step in setting up equations?
chessmaster7 2009-04-03 20:44:19
we need to find DH?
rrusczyk 2009-04-03 20:44:26
Sure. Let's call that x.
rrusczyk 2009-04-03 20:44:38
What else do we have?
mathq 2009-04-03 20:45:28
CH:CB=x:AB which equals CH:12=x:5
ThinkFlow 2009-04-03 20:45:28
FH = 2/3x
GoldenFrog1618 2009-04-03 20:45:28
x/5=CH/12
rrusczyk 2009-04-03 20:45:35
From DHF ~ ABF, we have FH/DH = BF/AB = 2/3, so FH = 2x/3.
rrusczyk 2009-04-03 20:45:41
From DHC ~ ABC, we have CH/DH = BC/AB = 12/5, so CH = 12x/5.
rrusczyk 2009-04-03 20:46:07
And what do we do with this?
ThinkFlow 2009-04-03 20:46:15
FH + HC = 26/3
tinytim 2009-04-03 20:46:49
so x=65/23
rrusczyk 2009-04-03 20:46:57
rrusczyk 2009-04-03 20:47:03
Now, we're set. How do we find AD?
mismatchtea 2009-04-03 20:47:43
pythagorean's theorem?
ThinkFlow 2009-04-03 20:47:43
pythag
GoldenFrog1618 2009-04-03 20:47:43
sqrt((AB+HD)^2+(BH)^2)
rrusczyk 2009-04-03 20:47:52
rrusczyk 2009-04-03 20:47:57
rrusczyk 2009-04-03 20:48:03
We could pound this out with the Pythagorean Theorem now. Anyone see an easier way to do it?
tinytim 2009-04-03 20:48:25
continue using similar traingles
chessmaster7 2009-04-03 20:48:25
similar triangles again? :)
rrusczyk 2009-04-03 20:48:35
Yep. Similar triangles to the rescue again. DXA ~ FBA.
rrusczyk 2009-04-03 20:48:45
How does that get us to the answer relatively quickly?
ThinkFlow 2009-04-03 20:49:12
pythag on ABF
e714212835 2009-04-03 20:49:19
BA and BF are nice numbers, so we can easily find AF, and use ratios
rrusczyk 2009-04-03 20:49:26
rrusczyk 2009-04-03 20:49:46
Now it's just arithmetic.
rrusczyk 2009-04-03 20:50:25
We could do it this way:
chessmaster7 2009-04-03 20:50:27
do pythagoras to find AF then multiply by AX or AB
ThinkFlow 2009-04-03 20:50:27
AF = 5\sqrt{13}/3
rrusczyk 2009-04-03 20:50:31
Or this way:
rrusczyk 2009-04-03 20:50:35
rrusczyk 2009-04-03 20:50:59
Here are solutions with the "extend a side" approach:
wangsacl 2009-04-03 20:51:03
Extend AB and CD, so both of them intersect at E. We get AE=10, CE=13, AC=13. By applying the angle bisector theorem: DE=130/23, DC=169/23. Then, we could compute AD using this formula: AD^2=AC x AE - CD x ED. Then, we get: (let's skip the rigorous part): AD=60\sqrt{13}/23. Then, we get p+q+r=096
wangsacl 2009-04-03 20:51:03
you could derive that from stewart's theorem
dgreenb801 2009-04-03 20:51:03
Full solution with extension: Let AB and CD intersect at A'. We have, A'B=5 and A'C=13, by the angle bisector theorem we have CD=169/23 and DA'=130/23. Now Stewart's on triangle ABC finds AD (a little messy but not too bad) is 60sqrt{13}/23.
rrusczyk 2009-04-03 20:51:25
(You can see why I didn't continue with that approach -- looking for a solution without the heavy machinery of Stewart.)
Wikipedian1337 2009-04-03 20:51:28
I did the problem using coordinates and found it much more straightforward than pure geometry.
rrusczyk 2009-04-03 20:51:40
There's another way to approach the problem.
chessmaster7 2009-04-03 20:51:42
theres another (more complicated) way to extend CD to meet AB, then inscribe a circle and find the radius to be 10/3 (which is BF) and find AD from finding AF and FD from there.
rrusczyk 2009-04-03 20:51:45
And another.
ThinkFlow 2009-04-03 20:51:47
Should I know stewart's theorem?
rrusczyk 2009-04-03 20:52:02
It's really not that important. But I would try to prove it for fun.
spelljack825 2009-04-03 20:52:04
trig bashing is quick and straightforward
m1sterzer0 2009-04-03 20:52:04
the trig solution seems pretty elegant as well (perhaps even moreso than the geometry approach)
rrusczyk 2009-04-03 20:52:12
Beauty is in the eye of the beholder :)
MathAndKnowledge 2009-04-03 20:52:26
just the derivation, because you can easily derive it when you need a cevian length
rrusczyk 2009-04-03 20:52:32
True: (about Stewart)
rrusczyk 2009-04-03 20:52:36
We'll take a 5 minute break and then knock off the last 5 problems.
Hullohihell0 2009-04-03 20:56:53
how much longer is this math jam going to last?
rrusczyk 2009-04-03 20:57:01
Maybe 75 minutes.
ThinkFlow 2009-04-03 20:57:05
Just looked at http://en.wikipedia.org/wiki/Stewart's_theorem. Stewart's theorem seems pretty useful to me...
rrusczyk 2009-04-03 20:57:22
So - so. I think that any math contest problem that requires Stewart is just a bad problem.
rrusczyk 2009-04-03 20:58:01
On to problem 11!
rrusczyk 2009-04-03 20:58:06
rrusczyk 2009-04-03 20:58:18
Where do we start with this?
charles.du 2009-04-03 20:58:56
|log(m/k)| < log n
geoffreywong 2009-04-03 20:58:56
log(m/k)
dgreenb801 2009-04-03 20:58:56
log m-log k=log(m/k)
rrusczyk 2009-04-03 20:59:13
GoldenFrog1618 2009-04-03 20:59:17
turn the abs value into 2 inequalities
m1sterzer0 2009-04-03 20:59:17
rewrite the equation for positive and negative (log(m) - log(k))
charles.du 2009-04-03 20:59:30
and then get rid of the absolute value
rrusczyk 2009-04-03 20:59:41
We have two inequalities to worry about to get rid of the absolute value.
rrusczyk 2009-04-03 20:59:47
What are the cases we have to worry about?
late20s 2009-04-03 20:59:51
get rid of the log as well?
rrusczyk 2009-04-03 20:59:56
We'd love to do that :)
rrusczyk 2009-04-03 21:00:14
Let's get rid of the absolute value first.
rrusczyk 2009-04-03 21:00:15
How?
m1sterzer0 2009-04-03 21:00:23
m > k and m < k
rrusczyk 2009-04-03 21:00:37
If m>=k, then what do we have?
tinytim 2009-04-03 21:01:11
m/k<n
happysunnyshine 2009-04-03 21:01:11
m/k< n
rrusczyk 2009-04-03 21:01:31
happysunnyshine 2009-04-03 21:02:05
k/m<n
funtwo 2009-04-03 21:02:05
k/m<n, k<mn
tinytim 2009-04-03 21:02:12
If k>m, we have m/k>1/n (from log(1/n)=-logn)
Wikipedian1337 2009-04-03 21:02:12
m/k > 1/n
rrusczyk 2009-04-03 21:02:15
rrusczyk 2009-04-03 21:02:41
So, we must have both m/k < n and k/m < n. Now what?
funtwo 2009-04-03 21:03:25
dangit i already forgot how i got m/n<k<mn
funtwo 2009-04-03 21:03:25
oh i remember it now. rearrange the first inequality to get m/n<k
tinytim 2009-04-03 21:03:25
so we have m/n<k<mn
wangsacl 2009-04-03 21:03:25
restrict the value of k
rrusczyk 2009-04-03 21:03:40
From m/k < n, we have k > m/n, and from k/m < n, we have k < nm, so the values of k that satisfy the inequality range from m/n to mn.
rrusczyk 2009-04-03 21:03:42
Now what?
mathq 2009-04-03 21:03:47
use the fact that there are only 50 solutions?
rrusczyk 2009-04-03 21:03:50
We know that there must be exactly 50 integers k between m/n and mn (exclusive, meaning we can't include m/n or mn). How does that help?
Wikipedian1337 2009-04-03 21:04:55
mn = floor(m/n) + 51
wangsacl 2009-04-03 21:04:55
floor(m/n)+1<=k<=mn-1, so there are mn-floor(m/n)-1 possible values of k
rrusczyk 2009-04-03 21:05:10
We can say essentially this without the fancy notation, like this:
rrusczyk 2009-04-03 21:18:19
rrusczyk 2009-04-03 21:05:39
(That's often how I avoid floor notation -- just make an inequality instead.)
rrusczyk 2009-04-03 21:05:58
All that says is "there are 50 integers from m/n to mn, exclusive".
rrusczyk 2009-04-03 21:06:04
There's nothing fancy there.
Wikipedian1337 2009-04-03 21:06:13
That's why I was stuck on mn = floor(m/n) + 51 during the test and couldn't go any further!
rrusczyk 2009-04-03 21:06:33
Yeah, notation can really trap you!
rrusczyk 2009-04-03 21:06:47
Going to inequalities is a good way to get away from the floor notation.
rrusczyk 2009-04-03 21:06:48
Now what?
tinytim 2009-04-03 21:06:52
We only need to try n=1,2,...,7 because m>=n
rrusczyk 2009-04-03 21:07:02
This immediately places some restrictions on n. Since m >= n, if n > 7, then m(n-1/n) is more than 52. So, we can only have n = 2,3,4,5,6, or 7. (Obviously, n=1 won't work!)
tinytim 2009-04-03 21:07:05
n=1 obviously doesnt work
rrusczyk 2009-04-03 21:07:09
That doesn't give us too much to test!
rrusczyk 2009-04-03 21:07:13
What happens for n=7?
rrusczyk 2009-04-03 21:07:56
If n=7, we have 52 > m(7-1/7) >= 50. Any luck?
tinytim 2009-04-03 21:08:00
no solution
Wikipedian1337 2009-04-03 21:08:00
nope
funtwo 2009-04-03 21:08:03
nothing
rrusczyk 2009-04-03 21:08:06
No solutions here; if m=7, then m(7 - 1/7) is 48, and if m = 8, then the product is 48/7 more, which gives us a number that is greater than 52.
rrusczyk 2009-04-03 21:08:12
And we just keep cranking down.
rrusczyk 2009-04-03 21:08:15
If n=6, we have 52 > m(6 - 1/6) >= 50. Any luck?
tinytim 2009-04-03 21:08:26
nope
mathq 2009-04-03 21:08:26
no solutions?nope
funtwo 2009-04-03 21:08:26
no
rrusczyk 2009-04-03 21:08:28
Again, no. Here, we see that m(6-1/6) < 50 if m = 8 and m(6-1/6) >= 52 if m=9.
rrusczyk 2009-04-03 21:08:34
If n=5, we have 52 > m(5 - 1/5) >= 50. Any luck?
mathq 2009-04-03 21:08:56
no again
chessmaster7 2009-04-03 21:08:56
no
rrusczyk 2009-04-03 21:09:01
Nope. m = 10 gives us 48 and m = 11 gives us a number greater than 52. No luck.
rrusczyk 2009-04-03 21:09:05
If n=4, we have 52 > m(4 - 1/4) >= 50. Any luck?
tinytim 2009-04-03 21:09:11
no
mathq 2009-04-03 21:09:12
no...
rrusczyk 2009-04-03 21:09:13
Nope. m = 13 gives us 48 3/4 and m = 14 gives us a number greater than 52. No luck.
spelljack825 2009-04-03 21:09:16
is there no better way than trying all the remaining cases?
rrusczyk 2009-04-03 21:09:41
I don't know -- I don't think there's a way that's easier to see than bashing this out :)
chessmaster7 2009-04-03 21:09:43
well trying the cases isn't necessarily what you would call "time consuming"
rrusczyk 2009-04-03 21:09:46
Exactly :)
rrusczyk 2009-04-03 21:09:54
If n=4, we have 52 > m(4 - 1/4) >= 50. Any luck?
charles.du 2009-04-03 21:10:19
we just did that
rrusczyk 2009-04-03 21:10:21
lol
rrusczyk 2009-04-03 21:10:27
If n=3, we have 52 > m(3 - 1/3) >= 50. Any luck?
funtwo 2009-04-03 21:10:38
yes! 3*19=57, 19/3>6
tinytim 2009-04-03 21:10:39
Yay,m=19
chessmaster7 2009-04-03 21:10:39
m can be 19
GoldenFrog1618 2009-04-03 21:10:40
19
rrusczyk 2009-04-03 21:10:42
Yes! m = 19 gives us m(3 - 1/3) = 50 2/3. We test that (m,n) = (19,3) satisfies the problem.
rrusczyk 2009-04-03 21:10:47
late20s 2009-04-03 21:12:08
7 - 56
charles.du 2009-04-03 21:12:08
7 through 56
andrewda 2009-04-03 21:12:08
7to56
spelljack825 2009-04-03 21:12:08
56>=k>=7
rrusczyk 2009-04-03 21:12:10
Any k that is more than a third of 19 and less than 3 times 19 satisfies the inequality. That gives us 7, 8, 9, . . . , 56. Indeed, there are 50 such k.
rrusczyk 2009-04-03 21:12:15
And finally, n = 2, which gives us 52 > m(2 - 1/2) >= 50. Any luck?
tinytim 2009-04-03 21:12:47
yay again, m=34
wangsacl 2009-04-03 21:12:47
34
rrusczyk 2009-04-03 21:12:50
Yes. Here, m=34 is the lucky number, since 34(2-1/2) = 51.
rrusczyk 2009-04-03 21:12:54
rrusczyk 2009-04-03 21:12:59
And our answer is ?
tinytim 2009-04-03 21:13:39
So the sum of all possible mn is 19*3+34*2=125.
GoldenFrog1618 2009-04-03 21:13:39
68+57=125
charles.du 2009-04-03 21:13:39
68+57=125
wangsacl 2009-04-03 21:13:39
68+57=125?
rrusczyk 2009-04-03 21:13:41
The answer to the problem then is 3(19) + 2(34) = 125.
ThinkFlow 2009-04-03 21:14:44
So we didn't like absolute values or logs so we found a way to get rid of them?
rrusczyk 2009-04-03 21:14:47
Exactly.
ThinkFlow 2009-04-03 21:18:55
next problem?
rrusczyk 2009-04-03 21:18:58
Good idea.
rrusczyk 2009-04-03 21:19:04
rrusczyk 2009-04-03 21:19:19
Any ideas where to start?
rrusczyk 2009-04-03 21:20:03
One way to start with this would be to try a smaller set of integers, like 1 through 5 or 1 through 7. Unfortunately, no patterns jump out when we try this.
rrusczyk 2009-04-03 21:20:07
We might also try constructing pairs in some smart way, but that runs into trouble pretty quickly. There are all sorts of strategies to take in making the pairs, and it's not clear which is best or why.
tinytim 2009-04-03 21:20:13
Try to find the upper bound of k?
rrusczyk 2009-04-03 21:20:21
We have a maximization problem -- we want to find the maximum value of k. Finding a maximum means that we need to show that k <= (some number), and that it is possible for k to equal that number. In other words, we need to build an inequality that involves k.
rrusczyk 2009-04-03 21:20:40
How might we go after finding an inequality that involves k?
spelljack825 2009-04-03 21:20:47
sum the sums?
funtwo 2009-04-03 21:20:47
consider the total sum of all the pairs
rrusczyk 2009-04-03 21:21:01
Can we bound this sum in any useful way?
math154 2009-04-03 21:22:39
For k sets, the minimum sum of the numbers in all is 1+2+...+2k, and the maximum sum is 2009+2008+...+(2010-k), and clearly the maximum sum has to be greater than the minimum for k to work.
rrusczyk 2009-04-03 21:22:55
We can bound the sum of all the numbers in the pairs. If we add all 2k numbers in the k pairs, the smallest the sum can possibly be is 1+2+3+ . . . + 2k, which equals k(2k+1). Letting the sum of the 2k numbers in the pairs be s, we have s >= k(2k+1).
rrusczyk 2009-04-03 21:23:07
Where does the upper bound math154 identified come from?
funtwo 2009-04-03 21:23:35
all sums must be distinct and <=2009
happysunnyshine 2009-04-03 21:23:37
all sums are distinct and less than or equal to 2009
rrusczyk 2009-04-03 21:23:41
Since each pair of numbers has sum no greater than 2009, and the sums are all distinct, we have s <= 2009 + 2008 + 2007 + . . . + (2010 - k).
rrusczyk 2009-04-03 21:23:49
Can we write that more simply?
GoldenFrog1618 2009-04-03 21:24:22
2010k-k(k+1)/2
rrusczyk 2009-04-03 21:24:25
We can write this more simply as s <= 2010k - (1+2+3+. . . +k) = 2010k - k(k+1)/2.
rrusczyk 2009-04-03 21:24:31
And now?
funtwo 2009-04-03 21:24:47
minimum must be less than maximum
ThinkFlow 2009-04-03 21:24:47
write our inequality
tinytim 2009-04-03 21:24:47
Combine and solve the inequality
rrusczyk 2009-04-03 21:24:53
rrusczyk 2009-04-03 21:25:00
And what does this tell us?
tinytim 2009-04-03 21:25:48
Solving for k gives k<=803
funtwo 2009-04-03 21:25:48
a regular quadratic inequality, k>0<804
rrusczyk 2009-04-03 21:25:59
Nothing fancy here. Just a bit of algebra.
rrusczyk 2009-04-03 21:26:04
rrusczyk 2009-04-03 21:26:15
rrusczyk 2009-04-03 21:26:20
Are we finished?
rrusczyk 2009-04-03 21:26:44
We have a guess. But on the USAMO, we wouldn't nearly be done.
rrusczyk 2009-04-03 21:26:50
We have to:
tinytim 2009-04-03 21:26:56
Now we must find 803 pairs that satisfy the conditions...
ThinkFlow 2009-04-03 21:26:56
no; that's a bound
charles.du 2009-04-03 21:26:56
need to find a way to use 803 pairs
#H34N1 2009-04-03 21:26:56
Prove that value is achievable?
tinytim 2009-04-03 21:26:56
we cant just assume that 803 works.
m1sterzer0 2009-04-03 21:26:56
need to show that 803 can be achieved
rrusczyk 2009-04-03 21:27:02
Not quite finished. We have to show that it is possible to come up with 803 pairs that have different sums. All we have shown is that it is not possible to have more than 803 pairs. (So, at the very least, we have a decent *guess* at the answer.)
rrusczyk 2009-04-03 21:27:06
Where might we start with building the pairs?
rrusczyk 2009-04-03 21:27:16
What might your first step be in trying to build 803 pairs?
rrusczyk 2009-04-03 21:28:09
If you were to write out 803 blank pairs, what numbers might you write as the first in each?
happysunnyshine 2009-04-03 21:28:55
1,2,3..803
GoldenFrog1618 2009-04-03 21:28:55
1-803
billy314 2009-04-03 21:28:55
1,2,3,...,803
rrusczyk 2009-04-03 21:29:03
We can start by trying to build 803 pairs in which a number from 1 to 803 is one of the numbers in each pair. So, the pairs are:(1, ?)(2, ?)(3, ?). . .(803, ?)
rrusczyk 2009-04-03 21:29:21
What are we pretty sure at least one of these sums will equal?
billy314 2009-04-03 21:29:43
2009
chessmaster7 2009-04-03 21:29:43
2009?
Bl00d 2009-04-03 21:29:43
2009
GoldenFrog1618 2009-04-03 21:29:43
2009
tinytim 2009-04-03 21:29:43
2009?
rrusczyk 2009-04-03 21:30:08
OK, we start with the extremes. Are we going to make an extreme low sum with 803?
Bl00d 2009-04-03 21:30:18
no
spelljack825 2009-04-03 21:30:21
no
rrusczyk 2009-04-03 21:30:25
Certainly not.
Bl00d 2009-04-03 21:30:27
so that would probably = 2009
rrusczyk 2009-04-03 21:30:36
Let's start with the extremes. We won't make a little sum using the 803, since we've already used all the numbers from 1 to 803. So, we'll make a big sum there:(1, ?)(2, ?)(3, ?). . .(803, 1206)
cool4ever 2009-04-03 21:31:22
figure out the sums
rrusczyk 2009-04-03 21:31:30
What do we think the other sums will be?
ThinkFlow 2009-04-03 21:31:45
sums: 2008, 2007, 2006, etc.?
GoldenFrog1618 2009-04-03 21:31:52
2008,2007,2006...
rrusczyk 2009-04-03 21:31:55
All the way down to what?
chessmaster7 2009-04-03 21:32:10
1207
charles.du 2009-04-03 21:32:10
1207
rrusczyk 2009-04-03 21:32:12
No sum can be greater than 2009, so we try to build pairs that will give us the 803 greatest possible sums. That is, the sums 2009, 2008, 2007, . . ., 1207.
rrusczyk 2009-04-03 21:32:27
So, we made the biggest sum with 803.
rrusczyk 2009-04-03 21:32:32
Can we make the smallest with 1?
ThinkFlow 2009-04-03 21:32:40
no
spelljack825 2009-04-03 21:32:40
no
charles.du 2009-04-03 21:32:40
no
rrusczyk 2009-04-03 21:32:48
Nope; we already used 1206.
rrusczyk 2009-04-03 21:32:56
So, what will we do to make 1207?
chessmaster7 2009-04-03 21:33:07
so we go to 2 with 1205
tinytim 2009-04-03 21:33:07
2,1205
e714212835 2009-04-03 21:33:12
2 and 1205?
rrusczyk 2009-04-03 21:33:13
We can't make 1207 with 1, since we've already used 1206. So, we use 2:(1, ?)(2, 1205)(3, ?). . .(803, 1206)
rrusczyk 2009-04-03 21:33:16
What next?
tinytim 2009-04-03 21:34:02
try to make the second largest sum? (2008)
rrusczyk 2009-04-03 21:34:12
How are we going to make 2008? With 802?
tinytim 2009-04-03 21:34:31
we cant to 802,1206 so we must to 801,1207
ThinkFlow 2009-04-03 21:34:31
uses 1206, so no
GoldenFrog1618 2009-04-03 21:34:31
no, 801
funtwo 2009-04-03 21:34:31
we cant. 801
rrusczyk 2009-04-03 21:34:34
We take another step, building 2008. We can't do so with 802, since 1206 is taken, so:(1, ?)(2, 1205)(3, ?). . .(801, 1207)(802, ?)(803, 1206)
rrusczyk 2009-04-03 21:34:42
What's next?
m1sterzer0 2009-04-03 21:34:52
back to the small end
tinytim 2009-04-03 21:34:52
then do the second smallest sum
happysunnyshine 2009-04-03 21:34:56
1208
e714212835 2009-04-03 21:34:56
1208?
chessmaster7 2009-04-03 21:34:56
now can we do the second smallest sum
rrusczyk 2009-04-03 21:35:00
Which is?
tinytim 2009-04-03 21:35:09
3,1205 doesnt work so 4,1204
chessmaster7 2009-04-03 21:35:10
4 and 1204 = 1208
m1sterzer0 2009-04-03 21:35:10
4 + 1204
e714212835 2009-04-03 21:35:10
4, 1204
rrusczyk 2009-04-03 21:35:11
Next, we make 1208. We can't use 3, but we can use 4:(1, ?)(2, 1205)(3, ?)(4, 1204). . .(801, 1207)(802, ?)(803, 1206)
rrusczyk 2009-04-03 21:35:20
Aha!
tinytim 2009-04-03 21:35:24
a pattern!
chessmaster7 2009-04-03 21:35:43
since we're going up along even numbers and down along odds, all the sums will work like teeth of a comb fitting together
rrusczyk 2009-04-03 21:35:47
And now the pattern emerges!(2,1205)(4,1204)(6,1203)(8,1202). . .(802, 805)(803,1206)(801,1207)(799, 1208). . .(1, 1607)
rrusczyk 2009-04-03 21:35:59
There is joy in Mudville.
rrusczyk 2009-04-03 21:36:00
We thereby form the sums 1207, 1208, . . . 1607,1608,1609, . . ., 2009, as desired.
rrusczyk 2009-04-03 21:36:09
The maximum value of k is therefore 803.
rrusczyk 2009-04-03 21:36:23
Now we're finished. (There are other ways to form 803 sums -- see if you can find some.)
rrusczyk 2009-04-03 21:36:44
But for now, we're on to #13
rrusczyk 2009-04-03 21:36:48
rrusczyk 2009-04-03 21:37:05
rrusczyk 2009-04-03 21:37:20
We could go after this with all sorts of trig. But . . .
funtwo 2009-04-03 21:37:23
this immediately reminds of roots of unity
rrusczyk 2009-04-03 21:37:42
Regular polygons. Already in a circle! This really makes us think of complex numbers.
rrusczyk 2009-04-03 21:38:00
How can we relate the problem to complex numbers?
dragon96 2009-04-03 21:38:24
the real imaginary plane
rrusczyk 2009-04-03 21:38:41
And how can we view these on the complex plane?
funtwo 2009-04-03 21:38:54
C1 is the first 14th root of unity, C2 is the second 14th root of unity...
tinytim 2009-04-03 21:38:54
Let the Center of the semicircle be the origin
charles.du 2009-04-03 21:38:54
let 0+0i be the midpoint of AB
rrusczyk 2009-04-03 21:38:58
rrusczyk 2009-04-03 21:39:14
rrusczyk 2009-04-03 21:39:19
ThinkFlow 2009-04-03 21:39:58
But how do we find the lengths of the chords?
rrusczyk 2009-04-03 21:40:16
Good question. Do we want to stick with the diagram we have up there?
hello123 2009-04-03 21:40:29
1-w^i
rrusczyk 2009-04-03 21:40:42
Will this work for the chords at B?
all4math 2009-04-03 21:41:01
no
GoldenFrog1618 2009-04-03 21:41:04
they are the same for B
rrusczyk 2009-04-03 21:41:12
The chords have the same lenth, yes.
m1sterzer0 2009-04-03 21:41:32
we can replace the chords from B with chords drawn to the lower half of the circle
rrusczyk 2009-04-03 21:41:43
Geometrically speaking, we can use this fact to make the diagram prettier:
rrusczyk 2009-04-03 21:41:48
rrusczyk 2009-04-03 21:42:39
We want the product of all of these lengths except for the middle one (the diameter), which is not included in the problem.
rrusczyk 2009-04-03 21:42:51
Now, how can we write that product of lengths in terms of complex numbers?
funtwo 2009-04-03 21:44:12
the distance between two points is the magnitude of their difference
benzi455 2009-04-03 21:44:12
abs of product of (2-2w^n) as n ranges from 1 to 13 but does not equal 7
rrusczyk 2009-04-03 21:44:15
The distance between w and z in the complex plane is |w-z|. So, we can write our desired product as:
rrusczyk 2009-04-03 21:44:19
rrusczyk 2009-04-03 21:44:31
all4math 2009-04-03 21:44:33
It looks cleaner
rrusczyk 2009-04-03 21:44:39
It looks a lot clearner!
rrusczyk 2009-04-03 21:44:45
Cleaner, too.
rrusczyk 2009-04-03 21:44:51
How can we compute this product?
xelalex1231030 2009-04-03 21:44:57
but it helps to add it in
rrusczyk 2009-04-03 21:45:26
Then what will the product look like -- that is, what product that we know how to handle will it look like?
xelalex1231030 2009-04-03 21:45:56
put that factor back in, and note that it's 1 + z + z^2 + ... + z^13 evaluated at 1
hello123 2009-04-03 21:45:56
consider the factored for of x^13+x^12+.....+1
rrusczyk 2009-04-03 21:46:10
Let's see where that comes from:
rrusczyk 2009-04-03 21:46:10
ThinkFlow 2009-04-03 21:46:37
it doesn't depend on w?
rrusczyk 2009-04-03 21:46:57
funtwo 2009-04-03 21:47:00
roots of x^14=1
xelalex1231030 2009-04-03 21:47:00
it's x^14 - 1!
rrusczyk 2009-04-03 21:47:08
rrusczyk 2009-04-03 21:47:21
So, how will we get from this to the product we want:
rrusczyk 2009-04-03 21:47:34
rrusczyk 2009-04-03 21:47:51
We want to jam in x = 1, but then we get . . . 0=0.
rrusczyk 2009-04-03 21:47:58
Hurm. That's no fun. What do we do?
tinytim 2009-04-03 21:48:16
divide by (x-1) first.
Yongyi781 2009-04-03 21:48:16
Divide by (x-1)
rrusczyk 2009-04-03 21:48:25
rrusczyk 2009-04-03 21:48:29
How does the right-hand side simplify?
Yongyi781 2009-04-03 21:48:46
1+x+x^2+...+x^13
rrusczyk 2009-04-03 21:48:48
rrusczyk 2009-04-03 21:48:54
rrusczyk 2009-04-03 21:49:08
So, do we have our desired product?
tinytim 2009-04-03 21:49:20
iremember to divide by 2
math154 2009-04-03 21:49:20
No, but w^7=-1, so 1-w^7=2 and we can divide this out to get the desired expression.
xelalex1231030 2009-04-03 21:49:20
but then divide by two because 1 - w^7 = 1 - (-1) = 2
rrusczyk 2009-04-03 21:49:28
rrusczyk 2009-04-03 21:49:32
rrusczyk 2009-04-03 21:49:39
rrusczyk 2009-04-03 21:49:48
And now we're finally ready to find:
tinytim 2009-04-03 21:50:05
So the answer is 2^12*7=672 (mod 1000)
funtwo 2009-04-03 21:50:06
7*96=672
rrusczyk 2009-04-03 21:50:11
rrusczyk 2009-04-03 21:50:34
Complex numbers are fun.
rrusczyk 2009-04-03 21:50:44
And now it's time for number 14.
wangsacl 2009-04-03 21:50:47
Is there any other approaches?
rrusczyk 2009-04-03 21:50:57
A wall of trig identities can get you there, I'm betting.
spelljack825 2009-04-03 21:51:04
much nicer than trig bashing XP
rrusczyk 2009-04-03 21:51:18
Yeah, complex numbers hide a lot of the machinery of trig in very nice ways.
dragon96 2009-04-03 21:51:24
its interesting how a geometry problem turns out to use the roots of unity
rrusczyk 2009-04-03 21:51:39
Whenever you see a regular polygon in a hard problem, this is something you should consider.
rrusczyk 2009-04-03 21:51:45
rrusczyk 2009-04-03 21:51:59
Where would you naturally start with this problem to get a handle on it?
MathAndKnowledge 2009-04-03 21:52:11
compute some terms
happysunnyshine 2009-04-03 21:52:11
to try some
ThinkFlow 2009-04-03 21:52:11
plug small values in
charles.du 2009-04-03 21:52:11
generate first few terms
rrusczyk 2009-04-03 21:52:22
rrusczyk 2009-04-03 21:52:31
rrusczyk 2009-04-03 21:52:40
rrusczyk 2009-04-03 21:52:51
You may be able to see some patterns, but the most significant observation is that each of the terms we computed is rational.
rrusczyk 2009-04-03 21:53:09
rrusczyk 2009-04-03 21:53:39
Any suggestions (assuming you don't see the magical trig substitution at this point)?
tinytim 2009-04-03 21:54:02
Take out the square root?
rrusczyk 2009-04-03 21:54:17
That would be nice, but squaring this looks ugly.
rrusczyk 2009-04-03 21:54:38
What's ugly about the expression in the square root?
rrusczyk 2009-04-03 21:54:46
(That is, what's hard to deal with?)
funtwo 2009-04-03 21:54:49
4^n
ThinkFlow 2009-04-03 21:54:49
n in an exponenet
xelalex1231030 2009-04-03 21:54:49
the 4^n
tinytim 2009-04-03 21:54:49
4^n
rrusczyk 2009-04-03 21:54:56
Yeah, the 4^n is no fun.
rrusczyk 2009-04-03 21:55:01
What might we do about that?
xelalex1231030 2009-04-03 21:55:10
we could divide out by it
rrusczyk 2009-04-03 21:55:19
Let's see what that looks like:
rrusczyk 2009-04-03 21:55:33
What does the resulting equation look like?
PI-Dimension 2009-04-03 21:56:22
8/5 a_n+6*2^n/5sqrt(1-thingy)
rrusczyk 2009-04-03 21:56:36
rrusczyk 2009-04-03 21:57:05
Hmm. Not beautiful. But not quite as scary. What might we do to make it even less scary?
xelalex1231030 2009-04-03 21:57:15
let b_n = a_n / 2^n
rrusczyk 2009-04-03 21:57:21
And why would we do that?
PI-Dimension 2009-04-03 21:57:41
get rid of the denominator
ThinkFlow 2009-04-03 21:57:41
get rid of n in the exponent
xelalex1231030 2009-04-03 21:57:41
then we could get rid of all the 2^n terms
rrusczyk 2009-04-03 21:57:42
Our goal here is to make the radical expression something we are less afraid of.
rrusczyk 2009-04-03 21:57:56
xelal's substitution gives:
rrusczyk 2009-04-03 21:58:19
rrusczyk 2009-04-03 21:58:30
Ugh. Now we have a's and b's. What do we do?
chessmaster7 2009-04-03 21:58:57
write everything in terms of b_n?
bbill955 2009-04-03 21:58:57
convert a's to b's
ThinkFlow 2009-04-03 21:59:02
express everything in b's. The 2^n disappears as well
facis 2009-04-03 21:59:02
divide by 2^n and substitute more b's in
rrusczyk 2009-04-03 21:59:04
We sub in for b for the other a's as well.
rrusczyk 2009-04-03 21:59:12
rrusczyk 2009-04-03 21:59:18
rrusczyk 2009-04-03 21:59:27
This is an equation you can almost take home to mom and dad.
Yongyi781 2009-04-03 21:59:30
Trig substitution?
funtwo 2009-04-03 21:59:30
looks like pythagorean identity
rrusczyk 2009-04-03 21:59:58
Now, the trig substitution becomes a little more apparent. But we don't need it to finish. What might we do now that we have a simpler equation?
benzi455 2009-04-03 22:00:15
crank it
rrusczyk 2009-04-03 22:00:34
We can start cranking out terms. We already have the first few:
rrusczyk 2009-04-03 22:00:38
rrusczyk 2009-04-03 22:00:51
What happens when we go after the next one?
ThinkFlow 2009-04-03 22:00:57
difference of squares!
rrusczyk 2009-04-03 22:01:06
This observation is particularly helpful here.
hello123 2009-04-03 22:02:20
what?
rrusczyk 2009-04-03 22:02:34
I mean, using difference of squares makes computing b_4 particularly easy.
rrusczyk 2009-04-03 22:02:40
What is it?
RoFlLoLcOpT 2009-04-03 22:03:34
$\frac{24}{25}$
rrusczyk 2009-04-03 22:03:37
ThinkFlow 2009-04-03 22:03:56
it repeats???
funtwo 2009-04-03 22:03:56
epic.
rrusczyk 2009-04-03 22:04:03
What does this tell us?
Bl00d 2009-04-03 22:04:05
wow
rrusczyk 2009-04-03 22:04:07
yeah.
xelalex1231030 2009-04-03 22:04:22
b_10 = 24/25 also
myrealname 2009-04-03 22:04:28
follow the pattern, you get b_10 = 24/25
rrusczyk 2009-04-03 22:04:33
It means that the terms b_2, b_3, b_4, ... alternate between 24/25 and 117/125.
rrusczyk 2009-04-03 22:04:42
Bl00d 2009-04-03 22:04:44
Could we have just cranked it out from the very beginning?
rrusczyk 2009-04-03 22:04:59
You'd have to catch the fact that the numerators are going up by factors of 4.
m1sterzer0 2009-04-03 22:05:02
it takes a bit of chutzpah to keep churning after that 117/125
rrusczyk 2009-04-03 22:05:10
True, but it's a recursion, and the AIME.
rrusczyk 2009-04-03 22:05:18
But the more natural approach is this:
charles.du 2009-04-03 22:05:20
let b_n = sin theta
rrusczyk 2009-04-03 22:05:35
Once you see sqrt(1-b^2), this substitution should come to mind.
rrusczyk 2009-04-03 22:06:00
(It makes finishing the problem a good bit easier if you're careful.)
rrusczyk 2009-04-03 22:06:18
(It's also a more powerful general technique that you'll use again and again.
Bl00d 2009-04-03 22:06:27
Coould we have just calculated a4
rrusczyk 2009-04-03 22:06:35
Sure, but the a_i don't repeat!
rrusczyk 2009-04-03 22:06:46
Remember, the b_i are a_i divided by 2^i.
rrusczyk 2009-04-03 22:07:08
Work out the trig substitution approach -- there's still a little trickery to work through there!
hello123 2009-04-03 22:07:15
so b_n+1=4/5*sintheta +3/5*costheta how does this help?
rrusczyk 2009-04-03 22:07:20
How does that help?
xelalex1231030 2009-04-03 22:07:45
sin addition formula
funtwo 2009-04-03 22:08:02
3-4-5 triangle
rrusczyk 2009-04-03 22:08:21
There's your hint. I'll let you sort out the details :)
rrusczyk 2009-04-03 22:08:33
One more problem!
rrusczyk 2009-04-03 22:08:39
rrusczyk 2009-04-03 22:08:57
Sigh. All those words.
rrusczyk 2009-04-03 22:09:02
We start with a diagram.
rrusczyk 2009-04-03 22:09:05
rrusczyk 2009-04-03 22:09:24
As with many good geometry problems, there are all kinds of first steps we can take here. There are many extra segments we can add, many angles and lengths we can find, and many angles and lengths we can relate to each other.
rrusczyk 2009-04-03 22:09:31
Before getting lost in a thicket of these observations, let's focus our efforts. Let's come up with a plan by focusing on what we need to find.
rrusczyk 2009-04-03 22:09:38
We want to learn about d, which is PQ. What are likely strategies for doing so?
rrusczyk 2009-04-03 22:09:45
For example, are we likely to be able to hit triangle PQC with the Law of Cosines?
funtwo 2009-04-03 22:10:25
well angle PCQ is constant
rrusczyk 2009-04-03 22:10:36
True. A and B are fixed, so <PCQ is fixed.
rrusczyk 2009-04-03 22:10:50
But why are we scared about law of cosines on PQC?
hello123 2009-04-03 22:11:07
PC and QC are variing
benzi455 2009-04-03 22:11:07
that's the only part of PQC that we know
rrusczyk 2009-04-03 22:11:11
We know that <ACB is fixed, which is promising, but we know very little about PC and QC, so the Law of Cosines doesn't look too promising. The Law of Sines suffers from the same problem.
rrusczyk 2009-04-03 22:11:29
What's likely to be our general strategy for getting at PQ? What are we likely to go after?
m1sterzer0 2009-04-03 22:12:00
MN - MP - QN?
e714212835 2009-04-03 22:12:00
MP and QN?
rrusczyk 2009-04-03 22:12:18
The most obvious remaining strategy is to go after MP and QN, and then subtract from 1.
rrusczyk 2009-04-03 22:12:34
Are we going to be able to combine them in some magical way, or should we go after them separately?
tinytim 2009-04-03 22:12:58
separately
dgreenb801 2009-04-03 22:12:58
separately, then add
ThinkFlow 2009-04-03 22:13:04
separately
all4math 2009-04-03 22:13:04
separate
rrusczyk 2009-04-03 22:13:06
We probably aren't going to be able to get at both of them at once, so let's focus on just one of them:
rrusczyk 2009-04-03 22:13:09
rrusczyk 2009-04-03 22:13:26
(In complicated geometry problems, redrawing a simpler diagram with just the pieces you're focusing on will sometimes help a great deal. Here, we know we can ignore AC, since MP does not in any way depend on A or C.)
ThinkFlow 2009-04-03 22:13:40
should be able to determine based on location of C
rrusczyk 2009-04-03 22:13:53
That's the goal. But how? What observations can we make?
myrealname 2009-04-03 22:14:04
cross-chord theorem?
tinytim 2009-04-03 22:14:04
power of a point
rrusczyk 2009-04-03 22:14:24
We have (MP)(PN) = (BP)(CP).
rrusczyk 2009-04-03 22:14:45
It's not clear what we'll do with that.
rrusczyk 2009-04-03 22:15:00
We'll stick it up there anyway in case it becomes handy.
xelalex1231030 2009-04-03 22:15:02
but PN = 1 - MP
dgreenb801 2009-04-03 22:15:03
PN=1-MP
rrusczyk 2009-04-03 22:15:10
We know that MN = 1, so MP = 1 - PN.
rrusczyk 2009-04-03 22:15:24
OK. We could combine that with the power of a point equation:
tinytim 2009-04-03 22:15:26
(1-MP)(MP)=(BP)(CP)
rrusczyk 2009-04-03 22:15:34
Not so obvious what to do with that...
rrusczyk 2009-04-03 22:15:36
Hmmm.
rrusczyk 2009-04-03 22:15:59
Stuck. On a geometry problem. What do we wonder when we are stuck on a geometry problem?
ThinkFlow 2009-04-03 22:16:13
What have we not used?
MathAndKnowledge 2009-04-03 22:16:18
what haven't we used yet
rrusczyk 2009-04-03 22:16:28
Exactly -- that's what we ask when we are stuck on a geometry problem.
dgreenb801 2009-04-03 22:16:32
we havent used diameter
ThinkFlow 2009-04-03 22:16:32
MB=3/5
m1sterzer0 2009-04-03 22:16:32
BM=3/5
hello123 2009-04-03 22:16:32
location of B
tinytim 2009-04-03 22:16:32
MB=3/5
rrusczyk 2009-04-03 22:16:51
Haven't used the fact that MN is a diameter, and we haven't touched the length of BM.
rrusczyk 2009-04-03 22:16:56
How can we use these?
hello123 2009-04-03 22:17:14
BN=4/5
rrusczyk 2009-04-03 22:17:34
Since MN is a diameter, BMN is a right triangle, and BN = 4/5.
rrusczyk 2009-04-03 22:17:46
Hmm. What else does that suggest we look at?
ThinkFlow 2009-04-03 22:18:10
MCN
rrusczyk 2009-04-03 22:18:16
And what do we know about that?
tinytim 2009-04-03 22:18:37
also right triangle
e714212835 2009-04-03 22:18:37
its a right triangle, too.
spelljack825 2009-04-03 22:18:37
right angle triangle
dgreenb801 2009-04-03 22:18:37
its right
rrusczyk 2009-04-03 22:18:43
rrusczyk 2009-04-03 22:18:58
OK, we now have some right triangles, and we know BN.
rrusczyk 2009-04-03 22:19:03
Um, now what?
tinytim 2009-04-03 22:19:32
cyclic quad?
rrusczyk 2009-04-03 22:19:44
True. Are we excited about breaking out Ptolemy?
tinytim 2009-04-03 22:19:54
no...
xelalex1231030 2009-04-03 22:19:54
not really
rrusczyk 2009-04-03 22:20:01
Ptolemy: (MN)(BC) = (BN)(CM) + (BM)(CN)
rrusczyk 2009-04-03 22:20:04
This doesn't involve MP or PN. No good.
rrusczyk 2009-04-03 22:20:12
Hurm.
rrusczyk 2009-04-03 22:20:34
Still stuck. The only useful relationships we have for MP so far are those we have above.
rrusczyk 2009-04-03 22:20:40
Those with MP and PN.
rrusczyk 2009-04-03 22:20:47
MP and PN . . .
rrusczyk 2009-04-03 22:20:50
MP and PN . . .
rrusczyk 2009-04-03 22:20:56
Tell me about those two.
ThinkFlow 2009-04-03 22:21:01
Which sum to 1
Yongyi781 2009-04-03 22:21:01
MP + PN = 1...
rrusczyk 2009-04-03 22:21:05
True. . .
rrusczyk 2009-04-03 22:21:14
What else is interesting about those two segments?
late20s 2009-04-03 22:21:37
the base of two triangles?
rrusczyk 2009-04-03 22:21:54
And what does that make us think?
tinytim 2009-04-03 22:22:10
area?
funtwo 2009-04-03 22:22:10
areas?
Yongyi781 2009-04-03 22:22:10
Area?
rrusczyk 2009-04-03 22:22:35
. . . Interesting. And how will we relate these two segments to area?
dgreenb801 2009-04-03 22:22:51
part of similar triangles?
rrusczyk 2009-04-03 22:22:57
They are part of similar triangles.
rrusczyk 2009-04-03 22:23:02
So?
rrusczyk 2009-04-03 22:23:08
What else are they part of ?
ThinkFlow 2009-04-03 22:23:11
They are a diameter?
rrusczyk 2009-04-03 22:23:21
They are part of the same *line*.
rrusczyk 2009-04-03 22:23:34
This leads me to one of my very favorite sneaky geometry strategies.
tinytim 2009-04-03 22:23:38
their ratio is equal to the ratio of the areas of the triangles
benzi455 2009-04-03 22:23:38
MP is to [BMC] as NP is to [BNC]
rrusczyk 2009-04-03 22:23:41
Whenever I have two important segment lengths along the same line in a tough geometry problem, I consider using area as a tool. We haveMP/PN = [BMP]/[BNP].
rrusczyk 2009-04-03 22:23:53
Do we know anything about the areas of these triangles that might be helpful?
ThinkFlow 2009-04-03 22:24:11
They add up to 6/5
rrusczyk 2009-04-03 22:24:16
True.
rrusczyk 2009-04-03 22:24:21
Anything else?
tinytim 2009-04-03 22:24:40
6/25 you mean?
rrusczyk 2009-04-03 22:24:43
Ya.
rrusczyk 2009-04-03 22:24:45
(We know the sum PM + PN, too. . . )
rrusczyk 2009-04-03 22:24:50
What else?
ThinkFlow 2009-04-03 22:24:59
proportional to MCN
rrusczyk 2009-04-03 22:25:07
What do you mean?
tinytim 2009-04-03 22:25:58
[BPC]/[PCN]=[BPM]/[MPC]
spelljack825 2009-04-03 22:25:59
MP/PN = [BMP]/[BNP] = [MPC]/[NPC]
rrusczyk 2009-04-03 22:26:02
We similarly have MP/PN = [PMC]/[PCN].
rrusczyk 2009-04-03 22:26:27
We don't know a whole lot about those two triangles. Hurm.
rrusczyk 2009-04-03 22:26:44
But these triangle area equalities seem interesting. . .
dgreenb801 2009-04-03 22:27:12
a/b=c/d means a+b/c+d equals them?
rrusczyk 2009-04-03 22:27:22
True; and what does that give us here?
dgreenb801 2009-04-03 22:27:47
[BMC][BNC]
rrusczyk 2009-04-03 22:27:53
We can combine the two area relationships we just found and note thatMP/PN = [BMC]/[BNC].
rrusczyk 2009-04-03 22:28:01
Is there anything interesting about these two triangles?
mathq 2009-04-03 22:28:45
share BC
late20s 2009-04-03 22:28:45
same base: BC
rrusczyk 2009-04-03 22:28:47
What else?
BenM 2009-04-03 22:29:05
related angles
rrusczyk 2009-04-03 22:29:08
How?
spelljack825 2009-04-03 22:29:10
BNC + BMC = 180..?
ThinkFlow 2009-04-03 22:29:10
M is the supplement of N
m1sterzer0 2009-04-03 22:29:13
angle M and angle N are supplementary
rrusczyk 2009-04-03 22:29:21
Aha! Why is that interesting?
dgreenb801 2009-04-03 22:29:33
area=(1/2)ab sin C?
benzi455 2009-04-03 22:29:33
same sine
m1sterzer0 2009-04-03 22:29:37
same sine
rrusczyk 2009-04-03 22:29:59
Ah, this might be an interesting tree to bark at; what do we find for MP/PN starting from this observation?
dgreenb801 2009-04-03 22:31:14
MP/PN=(3MC)/(4NC)
tinytim 2009-04-03 22:31:24
that is (BM)(MC)/(BN)(CN)
xelalex1231030 2009-04-03 22:31:24
MP/PN = (BM)(MC)/(BN)(NC)
rrusczyk 2009-04-03 22:31:54
rrusczyk 2009-04-03 22:32:11
Interesting
rrusczyk 2009-04-03 22:32:14
Why is this *particularly interesting* in this problem?
rrusczyk 2009-04-03 22:33:23
What I'm getting at here is that we've related MP/PN to MC/NC. MC and NC have *nothing to do with B*.
late20s 2009-04-03 22:33:49
cuz the other line share the same property
rrusczyk 2009-04-03 22:33:55
That means we can do essentially the same thing with point A to get an expression for MQ/QN in terms of C, not A or B. In other words, we can relate MP and QN, which is the goal in getting at PQ.
rrusczyk 2009-04-03 22:34:04
BenM 2009-04-03 22:35:36
Won't MQ/QN be equal to MC/CN
rrusczyk 2009-04-03 22:35:45
We do the same thing with Q:
rrusczyk 2009-04-03 22:35:46
rrusczyk 2009-04-03 22:36:02
ThinkFlow 2009-04-03 22:36:14
Because MAN is isoceles, right?
rrusczyk 2009-04-03 22:36:16
Right.
rrusczyk 2009-04-03 22:36:43
Now, what are we going to do.
rrusczyk 2009-04-03 22:36:49
We keep our eye on the ball. We want an expression for PQ that we can use. We have PQ = 1 - MP - QN.
rrusczyk 2009-04-03 22:37:03
So, what do we have to extract from those two equations up there?
hello123 2009-04-03 22:37:14
find expressions for PN and QN?
myrealname 2009-04-03 22:37:35
isolate MP and QN
rrusczyk 2009-04-03 22:37:37
We need expressions for MP and QN (PN and QN would be fine, too).
rrusczyk 2009-04-03 22:37:41
And how will we get these?
ThinkFlow 2009-04-03 22:37:43
Pn = 1-Mp and MQ = 1- QN
rrusczyk 2009-04-03 22:38:08
And what will we do about the right sides of those ratio equations to make this easier to deal with?
BenM 2009-04-03 22:38:25
make a new variable?
rrusczyk 2009-04-03 22:38:31
We can express both MP/PN and MQ/QN in terms of MC/NC. Moreover, we have PN = 1-MP and MQ = 1-QN, so we can express MP and QN in terms of MC/NC. That means we can express PQ in terms of this ratio. Let's give it a try.
spelljack825 2009-04-03 22:38:34
substitute a = MC/NC
rrusczyk 2009-04-03 22:38:39
rrusczyk 2009-04-03 22:39:06
Now we're getting somewhere. We find MP and QN:
rrusczyk 2009-04-03 22:39:09
ThinkFlow 2009-04-03 22:39:11
Now we can solve for MP and QN in terms of t, right?
rrusczyk 2009-04-03 22:39:14
Exactly.
rrusczyk 2009-04-03 22:39:19
And what do we do with these?
BenM 2009-04-03 22:39:31
substitute into that expression we had for PQ
ThinkFlow 2009-04-03 22:39:34
Subtract from 1
rrusczyk 2009-04-03 22:39:43
rrusczyk 2009-04-03 22:39:49
What do we do with that?
BenM 2009-04-03 22:39:56
simplify
mathq 2009-04-03 22:39:56
simplify
rrusczyk 2009-04-03 22:40:03
rrusczyk 2009-04-03 22:40:07
rrusczyk 2009-04-03 22:40:21
This is promising. It's a quadratic. The AIME-ized form of the answer looks like the root of a quadratic equation. We're on the right track.
rrusczyk 2009-04-03 22:40:26
But what do we do now?
BenM 2009-04-03 22:40:47
solve for t
ThinkFlow 2009-04-03 22:40:49
maximize d
rrusczyk 2009-04-03 22:40:51
How?
rrusczyk 2009-04-03 22:40:58
What do we know about t?
late20s 2009-04-03 22:41:32
oh, t is positive
BenM 2009-04-03 22:41:37
t is positive
spelljack825 2009-04-03 22:41:37
t is positive?
rrusczyk 2009-04-03 22:41:41
Do we know anything else about it?
rrusczyk 2009-04-03 22:42:18
What is t defined as?
dgreenb801 2009-04-03 22:42:22
t is a root of the quadratic
rrusczyk 2009-04-03 22:42:30
It is a root of the quadratic.
mathq 2009-04-03 22:42:33
MC/NC
tinytim 2009-04-03 22:42:33
MC/CN
ThinkFlow 2009-04-03 22:42:33
MC/NC
rrusczyk 2009-04-03 22:42:41
It is a ratio. This ratio is positive.
rrusczyk 2009-04-03 22:42:46
It is also . . .
m1sterzer0 2009-04-03 22:42:48
it can range from 0+ --> infinity
rrusczyk 2009-04-03 22:42:50
real.
rrusczyk 2009-04-03 22:42:56
Which tells us . . .
dgreenb801 2009-04-03 22:43:16
discriminant is positive!
MathAndKnowledge 2009-04-03 22:43:16
discriminant >= 0
rrusczyk 2009-04-03 22:43:32
The discriminant of the quadratic is nonnegative.
rrusczyk 2009-04-03 22:43:37
rrusczyk 2009-04-03 22:43:41
There was the equation we had.
rrusczyk 2009-04-03 22:44:18
What do we do now?
funtwo 2009-04-03 22:44:45
move the t over
rrusczyk 2009-04-03 22:44:50
MathAndKnowledge 2009-04-03 22:45:17
d^2-14d+1>=0 i think by plugging in the discriminant
tinytim 2009-04-03 22:45:17
D=d^2-14d+1
funtwo 2009-04-03 22:45:23
compute the discriminant in terms of d
rrusczyk 2009-04-03 22:45:29
This quadratic in t must have real solutions. That means that its discriminant must be nonnegative. We're set now.
rrusczyk 2009-04-03 22:45:33
rrusczyk 2009-04-03 22:46:08
And how do we finish this?
ThinkFlow 2009-04-03 22:46:33
draops 2009-04-03 22:46:33
quadratic formula
aggarwal 2009-04-03 22:46:33
complete the square
funtwo 2009-04-03 22:46:33
14-rt192/2=7-4rt3 so 7+4+3=14
rrusczyk 2009-04-03 22:46:43
Completing the square or the quadratic formula tell us:
rrusczyk 2009-04-03 22:46:46
rrusczyk 2009-04-03 22:47:02
What does this tell you about the quadratic d^2 - 14d +1 ?
rrusczyk 2009-04-03 22:47:43
Remember, we had the inequality d^2 - 14d + 1 >= 0
rrusczyk 2009-04-03 22:47:58
We just found the roots. How can we finish?
funtwo 2009-04-03 22:48:00
it cant be more than 1 the diameter and it cant be between 7-4rt3 and 7+4rt3 so the maximum is 7-4rt3
rrusczyk 2009-04-03 22:48:14
mathq 2009-04-03 22:48:21
7-4sqrt3 because it's less than 1
rrusczyk 2009-04-03 22:48:23
rrusczyk 2009-04-03 22:48:29
This was a hard, hard problem.

Copyright © 2024 AoPS Incorporated. This page is copyrighted material. You can view and print this page for your own use, but you cannot share the contents of this file with others.